Home
This site is intended for healthcare professionals
Advertisement

Finals Weekend: Sunday 9th January

Share
Advertisement
Advertisement
 
 
 

Summary

This medical training session will focus on medical knowledge pertinent to medical professionals. It will cover topics such as Upper GI, GI, vascular and basic surgical operations needed to diagnose and care for patients. It will be interactive, asking questions and discussing possible scenarios. There will also be teaching on topics such as abdominal, rectal and thoracic pain, medical investigations and common medical conditions. The presenter will also provide useful tips such as how to differentiate pain locations, how to justify CT scans, and common medical emergencies that should not be missed.
Generated by MedBot

Learning objectives

Learning Objectives: 1. Understand the different types of abdominal conditions associated with the right upper, left upper, and right/flank abdominal pain. 2. Identify common investigations and treatments of gallstone conditions, such as biliary colic. 3. Demonstrate the ability to differentiate between potentially life-threatening abdominal conditions, such as a triple A, ectopic pregnancy, and sepsis. 4. Become familiar with a range of imaging techniques, such as CT scans and ultrasounds, and when they should be used. 5. Comprehend the importance of performing a thorough physical exam and patient history to ensure an accurate diagnosis.
Generated by MedBot

Related content

Similar communities

View all

Similar events and on demand videos

Computer generated transcript

Warning!
The following transcript was generated automatically from the content and has not been checked or corrected manually.

okay. And then So your first the first and say is the Northeast is class. I would highly recommend coming to the Northeast. Rough way. I would also have the recommend meeting Scotland for various reasons in or teas of my own store. It's supposed the in the top three for a special training of you gave him his keep doing that Onda on energy will be starting off, I presume it's It's fine. It's not the bastard. Maroni's on the western world. It's all right. It's no worse than any other dominanted you come with me, right? So, uh, this strap is gonna be so focus kind of f y one knowledge. And I'm not getting into, like, loads of those detail about of actual operations, an actual surgery. It's more this kind of what you need to rest one which is and so being able to diagnose, um, basic stuff being up to spot generations underwent escalated, so it'll be a bit interactive. Otherwise, it would just be my voice which will send me to sleep on, uh, questions. Just interrupt me her mouth questions. Either it might the shop or just tell you my come and ask be a a. It's not exhausted, Ms. They'll be other stuff, you know? But you need to cover yourself because otherwise I'll be it'll three hours. So the kind of suffering that basically covers Upper Gi I G I some you're older stuff and vascular, and I haven't done any vascular ever. So if anything I've said, which is not quite right, and we're happy to be corrected by someone who knows about that. Vasser's have really done a big, big, big cash three centers, which I'm not that to any vascular thing. I talked to send them afternoon, that'll I'm not going to look at. Also call your thoracic prostate or pizza Already, some of them normal surgery stuff doesn't like to eat. That's if you need to do particularly to surgery. If you got especially orthopedics, you need to come today through. Do yourself right, say fasting beats or acute abdomen, and every kind of knows this diagram on day. Find it. This is the most useful way in an osteo situation exam situation, so you'd be able to differentiate what the dinosaurs might be based on where the pain is. So I'm trying that the child yes, and saying, starting from right upper quadrant. What if someone came in was sudden acute onset right upper quadrant pain. What kind of thing you thinking? Just be the same or pop it in the chart box? Yeah. Say thinking more or generally because you can't Diagnose comes to say it's clinically really kind of. Yeah, yeah. Say go bladder stuff. Never. Stuff like that's kind of way you can be looking for. Right question epigastric pain. What kind of music didn't that? Yeah, Gordon option pancreatitis yet something stomach gi. Yeah, that's about right. So peptic ulcer disease, Gastritis, pancreatitis. Good. Triple A. If someone comes in with abdominal pain, it's very unlikely to be triple A visit. Triple A, they'll be dead. Um, am I is another one you really don't want to miss, but back and often present in upper driving. I will come into that later. Left upper quadrant. Pain is really rare. It tends to be explained 30. Um, having it come. Oh, they're never bull house and driven already. Enough one and find Yeah s o rights or flank pain. We'll have country in pain. What? You kind of thinking. All right, I'm left rather, because it's the same in both sides. Yeah, something kidney. And so you got real college, which is stones. Then there's something else. Couldn't one of them. Actually, you don't. That's which is kind of all medical. Both often gets paid central words. Yeah, find the right ear's So you don't want to miss that? And then, uh, yeah, ground, that's all. Unbelievable. Pains or small bowel obstruction and possibly an early presentation of appendicitis that tends to be very generalized, right? That's also paying the classic. What would you be thinking? Yeah, you're You want to rule out appendix? Possibly an ectopic. Yeah, the That's the thing. Yeah, definitely. Especially in young women. You're gonna want to rule out sort of gynecology or obstetrics. Told you as well. Uh, IBD is a cover one is? Well, yeah. So boy I was gonna get out with this is I personally don't think this is usual. Because as you guys just pointed out to me, there are thousands of presentations for each individual thing. I don't think knowing where the pain is basically helps. And also, when you actually wet, no one can say I've got right here. Possibly it they just say I took the hurts, so I passed it. I think it is usually a big history is a lot more useful. But anything else? If it's like, Oh, you know, uh, I have my period and now I've got right right yet for sleep aid. Then you could be like, right. It's like a something like an X on it. Mrs. Use It was a starting point, but I don't get to take the usual going down the line, especially for our scale, right? So, yeah, things never to miss ethic actually paid you what that missus, do you absolutely want to make sure you don't mess? Yeah, yeah, especially in women. The reason I say that is women in tens off a much higher pain threshold than men. So women will often go to the GP saying they've got some congestion. But actually, it's a mist that I right away is planning on this one. It's a it's a nondescript pain. I'm just looking like absolute looking awful. That's you really want to make sure it's not the Triple A. Do you ever see a triple A and a CT scan? You you messed up and any wearing of childbearing age number, Script of the pain. Motion balancing What? What do you make sure you, Dave minutes. Excellent. Yeah, well, just Yeah, pregnancy. Just make very likely. It's probably pregnancy. I've seen a few of these left they So the action constant or any of those it might be on the headaches would actually that department right. Investigations and surgery Eso vessel be the bane of your life When you just start working a radiology date, just do scans because you also them. You have to kind of ask question of what we look for. And this is quite good. One to practice Torossidis as well that they'll ask why you Why do you want to do a scan? So bloods and everyone he does not get bloods done it. So is there acute bleed that, like also a modern media, is a renews or microcytic anemia in the elderly person? That's kind of nature. But does anyone know what that would kind of suggest? Yeah. Yeah, Colon, right. Americans. Yeah. So or just any kind of militancy? Pretty much. It's always done a rectal Um, yeah, these knees, eyes there. A renal dysfunction because of the vomiting so much into the knee. Replacement allergies are jaundiced. CRP is a sign of infection. So that's the one that's certainly really, really like it. Isn't that unusual postoperatively? Because it just goes up when you've had unoperated done and amylase and lipase These are blood tests for Well, yeah, So they they base about this or if you're suspected because you do Amazing, then scanning CT's are the best scan for surgical presentations. However, they are quite a big lack of radiation. So do you need to justify why you're doing one to be able to get it? So and also there's a lot of presentations that CDs either won't find or when really, are. So it's one that kind of needs discussing. Abdominal actually is only every useful kind of in one in one condition. What commission is that? Yeah, obstruction. I'm even then only and absolutely enormous obstruction that you shouldn't even that tray for so on the right chest actually is usual for a perforation. Yeah, uh, not about Don't watch. And I've done well, actually wouldn't really show it. It just actually write a raised right on the dot from for a perforation and then done world sounds. These a Macy's alerts on an exclusion tool, if you aren't sure, Diagnosis, which is more from the nuts. But if someone's no, actually, um, well, so this is kind of before appendicitis or something. Pelvic e or something. Really? Yeah. And you can also see littering. Little brother threw a note. Some notes. Well, they're kind of just died you to what's you should be doing and what we should be looking at for a CT rather than being a diagnosis to itself. Ground. Okay, say top it. One is go. Bladder stuff. Uh huh. Fine. So there are four so big commissions for Goldwater's that you kind of need to be aware of. So that is, and they go up and kind of kids and similarity so fast. It was big recall like, which is just just Goldstone's in the whole body with no information, no infection, it just comes. It's, like, sort of colicky pain. And Rapaport, it doesn't really cause any problems. Care. Cystitis is basically the step up from that where you have infected girl stones in the role butter. Uh, this thing quite easily dropped down into millions. Abscess which is really bad. Uh huh. But no one says that. Words they just say, See these names? But it's basically extending this combo duct, um, without infection on a seven cholangitis, which is probably the most of their one, it's presence off. In fact, it sends in the common bile. But I'm not when it's quite severe that one patients numbers aren't critic. Eight. You'd be on. Well, all right, so biliary colic, say presentation wise. It's right up according pain, colicky pain presentation. And it's emergencies negative if someone wants a either right or say how you elicit Murphy's sign because this is what way. If you say that, I'll see, they will ask you. I fell into this trap. Okay, Yeah, it's a kind of know. Maybe it's not more pain on my lap safe. No ST. It's when you be palpated on the right upper quadrant as they're taking a breath in it cause respiratory arrest. So, cause, um, like, sharply stop breathing because it's in so much pain on it's go. Actually, nothing to do with the left upper portion. Apparently, I always thought that as well, but it doesn't really make a difference, but The main thing is you take a big, deep breath out big, deep breath in, and as you're pressing down, they will stop breathing because they'll be nice. Uh, investigations. Wise me. Recall it. It's kind of a lot from blonde. There's nothing really there. You've been giving him. It's open up the rope, Sean. They don't really need a CT. Um, they're nothing. They're not back particularly well, so they don't need to stay in the hospital that in outpatient like only finds off strong pain medications, and they can come back and have a plan. That prescription thing is, is that you may have a little pre assessment stuff is, well right. Acute status, Say presentation wise. It's colicky. Rapaport, not Ted, didn't constant pain, generalized or infective symptoms. Fever, nausea, vomiting. So an Iraqi and not eating anything they're almost like the The matter is positive on your can be jaundiced. Well, depending on like the type of saying in the size of the state. So blood's under of the world. Sound is kind of your first line permanent because you could do a note sent out the bad side. You can see if there are like horrible concerns. Then your next goal. Some. Their investigation is an ML CP. This is not readily available. They won't do it, and I'll see P and 10 minutes where you This will often happen in the couple of base. So it depends how it usually a bowl. The patient is. If they are quite unwell, you can see see them. But unfortunately, eating doesn't really show much. Goldwater action doesn't show much bo Air Stones that already additions. So then I'll see you back for that management if it's an obstructing stain so that the stone that's blocking the combo. But I mean e L C p. It's not usually in the bones, not stop. It closes back. Them stands in the CBD. That's presentation. Wise is very similar, Teo. Very simple, very colicky, but most likely showed by a chemical. Joined us, say a bilirubin above, uh, can't remember above the thrush. Ah, my investigations. The abdominal ultrasound would really showed the gold start the goldstone it, but how about up? But I will show a big dialect. It's common bile duct, so that's kind of your indication. That additional statement that brought in the duct up, um now needing the LCP with the center on me. Men sending cholangitis is is the most of their one. They often presents off quite unwell. Hypertensive. Accept it, uh, that not exposed to it. Shock codes try, and I don't want to give me that. So this is the classic presentation? Yeah. Rival was in pain fever on and John this? Yeah, there is a pen topbas 12. It's the Shockers drive isn't even really a thing. It tends to just be so It's right upper quadrant pain with that. Quite a well as well. Bicycle right Investigation is only 10 ml see, because you need to see exactly where the stones on on that down the emergency. Also be right, because that's what much all things goal stains and Goldwater's. And that's pretty much heart of a large answer. She and this is the old letters they are completely annoying over and they keep getting infected. People get really sick of them. Topic is other upper gi I stuff. So the main room that your guts and upper GI stuff's been surgeries packets isis So severe abdominal pain epigastric region radio into the body generally, um well, just feeling like absolute crop know, eating anything. Investigations wise, um, malaise in a keeps. It needs to be greater than 300 for a dynasty. Pancreatitis, like pays tends to come up if it's more acute on chronic presentation or a sub acute presentation, and then they could get a CT. Pancreas is showing big information of the pancreas. Um, say causes. And does anyone made the acronym is that I want to give me some causes of pancreatitis. Yes, I get smashed. Is the Is the okay? Yes, I go stages. The GI is afternoon. And the rest of kind of, uh yeah, they're scorpion bites. Malignancy. Actually known malignancy doesn't tend to cause pancreatitis. Pectinate. That's what people billings. It doesn't tend to cause pancreatitis. My oxygen. It it's the eye is not so at Tradjenta that she needs Oh, is actually one. Because because it should be idiopathic. It should just come on anyway. Say steroids is a big one. Loads of Johnson called that, but most important, one is those things go. So is the one that will cause pancreatitis. Alcohol are holidays. Pancreas is not essential presentation. It's a medical preservation. So it shouldn't go to a special world There you are and scope in science. Yet they do technically called pancreatitis. I'm ready to find a surgeon if he's ever seen one on B E. R C E p Paste, the ercp pancreatitis can also cause it. I've only obviously not once, but that's pretty that religion and then scoring. The acronym is pancreas with anyone vaguely know what the uh, what their ranges are, what we get a note for and also just to point out there's two glasses. A score says the last, um, memory score, which is the pancreas moment. The glass a black should score, which is for upper gi bleeds, say, my former colleague, Skate 100 behavior and grounds me a score of someone off nine on my Publix because that would mean they have 100% mortality rates when actually, it's people's a glass of black streaks for a day. Say it was eating the wrong school Be. Actually, Ph is the last one. I'll just bring it up. Actually, yeah, it's not really that important. Your areas more trouble, but yeah, they actually in's That's aged neutrals being really high hypocalcemic. But if there were a renal functions. Guns part well, liver Know what Let her cry Files got a bit awful. Argument is too low on their hyperglycemia. I don't know if you need to make Bally's beds. It's a good one today because I can see some very, very grumpy G I consultant doing You're off the ass. And this eso the more important thing is you need to do specifically allergy to do a lipid profile to do an arterial blood gas on someone with kind of should be hungry with severe pancreatitis, you apparently get Cullen sign and great tennis sign. I've seen a guy in, I mean, who was being a 90 for nine months with necrotizing pancreatitis, and he didn't have either of these. So I would take these with the picture. So they're good ones to have to. You're asleep. But a day that you that they then they're real thing and say, You're so glad that last a practice going to predictable talent. See, this is very my pastor. You have it. It's not really on a percent mortality to seven or eight, but if it's anything more than five, you need to have a phone. I see you because they will most likely need to get to work for you because there are very, very scary everything the magnetize is to send you like it's the weirdest patients is that it's like your pancreas has been genuinely burnt and it's just passing out fluid. So you haven't been really, really descended. Tons and tons and tons of fluid inflaming ALS the surrounding show. You'll get dehydrated very, very quickly. So, yeah, it's necrotic belts form pseudo cysts, which can bend, become boule a which can also burst. So it's it's it's not a nice disease at all, and then management's so most important thing is true. Result station been a dietician, and that's primarily because, and they're not gonna have any symptom weeks, so they will need to replace all that size and optimized everything else don't need from a personal axis. Why do they need really, really shown pain relief for a package? I said what can happen on top of thing because that will make it gets even even worse. Yeah, and all the s so negligibly spiritually arrest. Yeah. So a, uh, the second drug offenses is quite a common. It's quite problem Issue that long term management. When pancreatitis schools information some dye down in May. Need to bend deal with the underlying issue. It's alcohol with accidents. If it's Goldstone June and fix a Little Stones right by the Upper GI I presentations. I'm going against too much detail, but just quit crying Questions. August payments Doing this in an older person. What is Agnes? Until proven otherwise? No. See unless they have. So it's a good question. But yes, it's like a gastric cancer. But just know about 200 pushing their do you give antibiotics or pancreatitis know you don't unless they have super added colecystitis, which is quite common because the girl said she called spine gets like this. If they have bathtub, you'll do. You treat that does is like this. Or if they have a pneumonia on top of the pancreatitis, which is very common as well, you're been given to the other, but not for the pancreatitis itself. One of this, and knowing things like that when I was from surgery, is like once is every shift I was out of hours. I would get friend by and they're saying this one pack if I spiked a temperature, little bit receptor. But pancreatitis is a really heavy inflammatory condition. So it will. It will cause you, despite temperatures that will cause you to almost look fevers. And I'm well, but it's privatized. Isn't function, is order. Okay, just turn this in a younger person. What? It wants to diagnosis. Go get back to nobody. But can you do call certain unity, setting a few steps? So, um yeah, I guess it yet it's Gilberts. Sorry, just one second. Sorry about your mother. If you could do a closest that, But can you do it so you can do it in Costa, slightest? Sending her lungs right? It's the issue. Is you go and do a acute a lot because it's a stone in the goal, But it's a high risk of slipped in the stain on back, because pancreatitis So you're more likely about want to do any Alsop internship trying? I got the same house rather than gonna take the gold brother out. And also with the sending her lung doctors that normal. So, um, well, they're not optimized with it, so that would be an emergency like, urgent procedure, which is kind of it comes with its own rest side. But it's not really a yes, and I wanted to That one. It it's ah, it's one that needs to be discussed by to you anesthetics and Jensen. Yeah, it was Ah, hot lot. So hot. Luckily, is it depends how well they are if they're really unwell, that for with colon cystitis then know on emergency, luckily would be your goal standard if they're not. But, um, well, and they can email you once once it then. Yes. So hot, hot, like earlier is you got some good. Yeah. All right. Yeah, Number right. This agent and I haven't gone into this too much bit because I've lost my past meds. That's destruction. But there is a really good past my page on this on there on the past textbook being on swallowing difficulties. And I would highly recommend reading it. You'll get one question finals about it, and it's very useful. Hematemesis. And what What's the likelihood? Ishan that comes with, like, acute hematemesis. Yeah. Barris is Yeah. So Yeah. Mallory Vices short pet. Yeah. I suffer job. Aris's you need You need to, um never imagined basket and it's called wrong call industrial money to don't like it. It's been in probably in any Actually. You shouldn't really see this in surgery. You want to say read Anne, Right? Okay, that's very much everything I have Upper GI I there Any other questions about Upper GI I stuff before we don't say colorectal. Next coupe. Fine, Cardell, you don't really need to know this. I just think it's quite it's quite using. Teo have an idea when you start working, people have, you know, long satchel names here prepped for surgeries. They probably never, you know what I mean. So this is quite good Repaid. That, I think is, is in the same side. You see, it's it's changed. You kind of what surgeries that hard on. What can be an estimate is on what can be joined in what can be pretty much the only two that can't be the anything that can be going on low anterior abdominal prone. Your reception's and take your part to Colectomy. Everything else can essentially be an estimated, even like take a colectomy. It can be nasty, most on the most common surgery of the heart mons, which is erect the signal colectomy with soap information on D reversal done at a later date. So it's essentially a rectal sigmoid club to me. Okay, correct presentation. So yeah, which all right dot um, what's that? Um, so, uh, car accidents? Do you actually, today you probably don't need to know any to be honest. Um, I just think when you start working, it's a particularly useful one to have on if you go into a nasty and it says what operation? They've had its use. It kind of vague visualize where they have and relationships for a stronger If there's any obvious, so lbs it. It happens when with pancreatitis, you have a chest infection because you're not taking deep enough inspirations because the pancreatitis is so sore. You're taking very, very shallow breaths, which can cause both eight Alexis on do a pneumonia. Which convention cause respiratory arrest or a RVs? So giving them strong opioids on really strong pain. Relief means that it kind of balances out the and the pain from the pancreatitis, and they can take deep breaths and they can do the chest physiotherapy. Now I know what you're going to say. Which is very strongly to its course. respiratory depression, so you kind of have to It's kind of about Expect you wouldn't just give him coding You give them. Like I would give someone mama's idea. Morphine. Give it regularly. Is that all right? The tea that resources questions, Coop. Right. Say case one in colorectal. So 25 female one day history of severe abdominal pain started around the umbilicus Move to the right. Yes, sir. What kind of things you thinking? The differential I went. This is a lot because I was taking up to this time. It's yet. Yeah, Very like May of emphasize. Make sure you exclude pregnancy. Is 25 year old female? Yeah. Make sure you're on Exelon. Yeah. Investigations. What investigations you gonna do? Yeah, Bloods depends home. Well, they are a f B C crp. If that Quite a billion. They're straight for a CC. You're in Austin. Very good proxy. That's very good. Um, imaging, it depends on Well, they are if they're pretty. Um, well, you probably go straight. The CT. So pentacel itis typically starts in like a pain which migrates the radio fossil they present generally. Well, it's most common in young people. It's not really common in older people that are absolutely essential with appendix that was wild. And what's wise you have racing from the market is a high white cell, a high CRP, but everything I should be more or less normal old style abdomen or a CT abdomen. They're both actually pretty reasonable to us or in this situation. But it depends on well, they are on. If you have a GI, it could be something else. Management. They need a lab and X. Um, but this this is quite common thing that happened on a special wards. This. So while they're waiting for their while, they're waiting to their scan. They suddenly become septic. Their BP tanks, their heart rate skyrockets. They look awful. That sweating, been vomiting. What are you going to do about point? Yeah, so except it's sexier. Know they'll already probably be on the buses and fluid at that point 80. Yeah. Uh huh. Yeah. Emily's like Byetta. You're just taking straight theater for a diagnostic lab. So whether it is an appendix or not at that, when you're probably wanted to stay in shape better so that either if they look back and well, it's either an appendix that's past or something else is even west. You don't miss it, you know, just go straight to say it. Uh, okay, let me see. Patient Generally severe abdominal pain, writhing around the pain, global tenderness there. Passing stool, maybe some diarrhea. Maybe some people bleeding. Not really into give you so much history because they're in absolute agony. Two main differentials that are serious that you want to rule out. And you need to make sure you'll these two out on Do what? To differentials a path. Yeah. Keegan. Yeah. So parents like this is itself not really a conviction. It's Ah, it's a secondary to So you're Optima just being inflamed. So perforation is one the other one is so yeah, Triple A. Most of the medicines are extremely a Yeah s Kenya is stay with visuals most Oh, hold on. Ah, so some just going back to the previous case? Um, it's stable, but not visualized. Not Sanders it clinical dinos and see So it depends on the case. It definitely isn't a stupid reason. Terrified of the night. It's necessary. God, tonight this is a very, very specific thing where someone has a viral illness proceeding the onset of abdominal pain. So typical one is it's a young guy coming in with a source. We had a sort of A for a week and now has a donut pain back to the generic of the night. This and if it's not visualized? Nope. Sounds, but that's still in pain. But they're stable. You said if the CT on at about points and did you was there that that is, uh, you probably you do it as a invasion. Telling you do is invasion. But, yes, there's some. If you can't see it on the outside, you probably do a CT. I never see. It is not showing something when you need to consider whether you want to do it down there. Still wrap, because if there's clearly something not, well, not loves, they're a bit screwed, then you'll probably still want to do it, but you just don't know what it is. Is that all right? Right. So you have the two things you want to make sure. Here you don't mess our senior on the past so you don't need surgeon. See to basically say I don't move. Preparation risk factors. The main risk factors for a pass are also on my body. The rest tunnels of the and quite high risk factors. That also is an IV. The upcoming, the more important ones previous ercp is it actually quite common is post theosophy perforations. So they're monitoring is the only access from the fluids and the antibiotics ABCDE, sepsis stuff, high dose PPI. So what we give is IV pantoprazole I know need a laparotomy magically and then skin it. Now, um, what's the number one risk factor for the skin? It? Well, no, it's not a half f is the second round trip. Yeah, smoking. Smoking is the number one risk factor. F m. I struggle. Heart defects is the second, and vasculitis and I procrastinated states is another risk factor bit. The main thing is that if someone smokes cigars cigarettes for 20 years and they come in, the general is a little weight. Think it's healing? Well, 100. They needed a frenzy, like brought to make sure. Um, you have a call about was dead and they'll need it if they come in on the night shift. This is something where you have to find the consult. Consult were coming to do it overnight. Um, I've seen the three of these presents at once. It was absolutely terrifying. Like case three, uh, 70 year old man presents last sided. Abdominal pain, diarrhea, Possibly some PR bleeding is not really sure. Raisins. Um, Jemarcus. He's kind of generally unwell. What kind of thing you thinking? What? What could this be? Well, yes. Yeah. Diverticulitis is the remains of differential here. Investigations, lives. What would you do for them? Blood. You try and do a CT. Although see, he's at least two miles were very reluctant to do. Yes, you're right. You need to get PR cities radio. Just have very reluctant to do CT scans for diverticula diverticulitis is because you can't really see them on the CT and the clinical diagnosis. Management wise, I Any ideas would be sure. Diverticulitis? Yes. So don't eat for two days. Why does it pr how to be honest? It probably doesn't had. That's if you don't do a PR and you've missed something like a rectal tumors and and you will get sued. You know, you'll need to do a PR. Um, I've never found a PR to be particularly useful to be honest, but I guess it kind of shows you how much blood there is around the rectum. If if it's possibly a hemorrhage drug about a a true diverticulitis, right? So that's your eyes. So that's kind of three tiers of diverticular diverticular, which is not inflamed diverticulitis, which is on inflamed outpouching. I'm betting you can get by particular abscesses, which all serious. I'm need emergency surgery so it will be less or a lot of pain, with some PR bleeding feeling generally well. Risk factors. The classic Add Enbrel slash stopped and diets of three D five miles a day. High fiber diets high fat by It's Pentecostal, and it's essentially just any a your bowel because well, like about your you're stressing about trying to paws out horrible, horrible fate. Manager in its tends to be conservative. Um, and you can do a colectomy if they are extremely on. Well, if I'm that you can do so obsessed monitor. If there is an abscess, it's you can typically now just is absolutely right. Takes four very similar presentation, but in a 25 year old so what kind of thing you thinking at this point Yes, I'd be. And also colitis on currents. So very similar. Your management here is not research. It's more gastric medicine, but lots of lots of steroids, basically. And then you discussed about colorectal MDT to see whether the colorectal surgeon will be a purpose. The one thing I would say is that the age of people getting bad verticulitis is coming down quite significantly because our diets just generally getting worst. So I've diagnosed a 30 year old with diverticulitis. So it is worth bearing in mind that while the age thing is a thing for exams, when you actually start working, it's a bit more cycle case life. Um, a 70 year old presented intermittent abdominal pain, constipation, distention and vomiting. What are you thinking here? This is one of the mill common sagittal presentation. So, yeah, extraction. Possibly secondary to have a rectal cancer. Yeah. So what other questions you kind of want to ask? So this is where you can You want to try and see what kind of surgeon this is? That's like you lost weight? Yeah. Weight loss. Probably then change in bowel habits. Have you had any previous surgeries there again? Well done. and investigations wise. You want to see ti them pretty? Actually. Imagine wise eso immediately you do a urine stuff decompression on The main reason you did this is you'll insert riles to because it's actually like a a blockage in the bile Cialis. She's gonna drain everything that's in the stomach and decrease the abdominal distention, and then you need to take for emergency surgery at the point of the transition. So where the blockage is, so this could also be managed. Laproscopically actually. So yeah, small bowel obstruction. Best is large bowel obstruction. This is kind of this is baby important. It's a very classic solve. MCT brushin. And so what are the team causes of a small bowel? Obstruction? These account for maybe 95% off small bowel obstructions. Tiny isn't infusions. Yeah, Volvulus is really, really, really rare. It's it's seen it once, and it was in a child. It's it's It's very, very right. Yeah, adhesions and things that counts and 95% than the primary symptom with a small bowel obstruction. Yeah, is vomiting that management strip? It's up from bed to surgery. This can actually often be managing without surgery if you just decompress the cancel self results, but if that happens, it does tend to help it again. March model structure in. And what is your primary causes of a large bowel? Obstruction? There's kind of two main ones. Yeah, concerts one. And then they're kind of similar. But like a stricture secondary to ah, a benign condition like Bob supplies refugee. So your primary center of a large bowel obstruction so promising tomorrow. That's what attraction is bombing thing. What is it for a large bowel obstruction their constipation? Constipation is mostly for it. It's It's more pain in the sentient distention is the main one on pain. Vomiting is not really think if someone's been working with a large bowel trouble, speaking of vomiting, and that's a really, really, really, really bad sign and they only to go to bed immediately. Management is very similar. Ah, dripping socks, surgery. Often people with a large bowel obstruction on knowing cancer, and they've got colorectal metastases. So then they're not going to be fit for surgeries, and then you can do things like stenting, and that's like France. Too much detail, full ones. I'm not gonna go into this, too, much of it. I think you guys probably do need to know this is the pee on on on the dukes staging for colorectal cancer. I have to admit, I have never, ever, ever used Jeeps. I only every news TNN, but I apparently adding the light juice. So, um, yeah, I would I would. Then these and then? Yeah, that's what everything colorectal is. So you said management off Alba. Yeah. So about preparation, you'll need a high dose PPI before century on. Then you need surgery. Yeah. You need a day for the past. Um, needs don't eat in the morning to like Broadway. Okay. Is that all right? Does anyone else have any other questions? Don't give antibiotics for a passion. Is that Yeah. You would. Yeah. Personal. Active when you get on both sides. So what you got? I think I think it's because on the risk of another perforation, if they perfect once in the in the stomach, the likelihood is there's lots of little ulcers that number one will. So you want to try and reduce the acid content, and I have the other reasons. Wells, if you reduce the acid content, it's gonna cause last damage to surrounding structures because it's your stomach. So I'm going to be passing out tons and tons of acid. The other thing is, all they need for a half is TPN or title parental and nutrition, because they will be able to eat because they'll be no one else. Is that all right for Colorado? Yes. Okay. Urology, Um, I know we've ever about urology because, um, uh, anyone who's being in the borders has had the pleasure of having Mr T here. Cameron teach urology on I can't get them every other year Old justice as weird as that man. Um, Right pace. One urology, 52 year old lady presents with right sided morning to burn pain comes in waves, Feels nauseous. Plus, plus the blood of your analysis. What you do, Correctol? Yeah. What made then you thinking? Yeah, it's not very tall. It the renal colic. They all being all different. So Yeah, it stayed kidney said, uh, investigation. What's the What is the gold standard investigation for all this seeds? Davey? Yep. Brigade management's, uh, complicates it all kind of bladder minutes. Yeah. So nephrolithiasis for a kid. Renal stone typically keep colicky more into growing pain. It's describes in multiple medical textbooks as the worst pain anyone could experience except shell birth on. So, your diagnosis for a with a CT can you be and then your management for it? What's your best analgesia for this? This is a typical MCQ question. Yeah, Diclofenac appeal. Yes. Very good. And then hydration if their infection markers are affected. How the Super Office Sobrino Infection. Um, this is kind of nature, but eh? So what will the sides of how quickly you need to take the surgical manager? But this one, if you've got a CT, can you be especially? No, an a k I actually wasn't doesn't typically affect it. Position, site, size, kind of position is the more important thing. So any sign of obstruction? Yeah, that's right. Leaving family. Yeah, obstruction will be on your factor whether this needs to be done immediately or overnight. And so I didn't You're only on my list of things that I'm supposed to whenever someone comes in with it. Obstructed state. I have send them off to do counsel. Right? So management's off renal and your socks things. So if there's any signs of obstruction. So there's hydronephrosis. Or you could see that the stone is blocking the D you opening. They need Sergent Surgery, so that can either be a stent or percutaneously for us to me. So you basically just to relieve the pressure in the kidney. So obviously a stent. They go in through the urethra, and they basically just put a tube in the in the ureter, that is, that's blocked. I don't trust me is an interventional radiology procedure. Whether it is going through the back on, just Dream the kidney. Well, if there's no evidence of retraction, Bennett's then as you guys have said, it's dependent on size. So they all get, uh, medical management. Hydration analgesia some antibiotics. Watch your weight. A. A swell takes time to be purple Actiq. If it's a distal, you're a terrorist own. You can try medical expulsion therapy with tamsulosin and of it's small was on your court. If that has come from it on, uh, just the Paris eats more and Kadian and morphine. Yeah, just standard pains. Okay? Yeah, pretty much. There's not really and I gargle won't go on it. Yeah, um, pretty much. Yeah. This part is well, codeine and more things. This is your cervix and invest in 10 movies is you can try medical it smokes in therapy is really what we Well, we could try it. And satchel intervention. Uh, so if it fails to pass spontaneously less than 10 minutes 10 mills, insides. You conduced AQ wave lithotripsy and then standing If that hasn't worked, um, if it's 10 20 mils, then you would go for a stent. First line on If it's bigger, you can go for pectineus left. Uh, that, um it's essentially where they go into the back again. Interventional radiology procedure. And they try and fish the stain out from the from the back through the kidney. It's a pretty wild decision. I've never really seen it done. Bit bad thing. If it's an enormous stone out of stock on stone, that will cause unbelievable amount of damage and it tries to pass out. Is that all right for renal stones? That's pretty much everything I have. Right? Cool hematuria. So what would your differential diagnosis be for someone who has a mature is coming with unity area? Yeah. Yeah. So ngp land Someone has hematuria You want Oh, you will. Someone for a fraction cystoscopy. It's a two week, right? Yes, UTI in Austin is really common most, and it's probably the most common. One is. The stones can cause it as well that cut bladder carcinoma, and you're only you're a few because you're a failure. Cancer them is, are the second most common. The other one to bear in mind as well is traumatic catheterization. If you Castro someone and then they start, you start passing immature. It is actually liking, caused by the catheter going on anything else and hematuria it unless they are genuinely hosing blood. Hematuria is never on emergency, so nurses get very, very twitchy about hematuria, and that really knows about it. But it was never an emergency management for it. If it's Frankie Materia, then you need a three way catheter. Why, What? What is the risk of having Frankie Materia? Yeah, court retention? Exactly. Very good. And then it It's just really a wine color, as it's beautifully name and management wise is, let's see, just fluids and trying to treat the underlying cause and watch. If it becomes frank, then you'd like investigate about the book honestly, been raising what rose? A wine cooler. You just leave it on. Then they'll go for a nerve agent systems to be another patient if they require it. If it's if the history says, Oh, it's probably something else, then you don't really need it Just taking the swelling. So this is quite on, and this is quite important. One. This comes up a fabulous. There's like four different types of swellings internally to be aware of. Um, I'm that's know mentioning about Kansas is woke. So number one hydro seals. So these trans illuminates they are. They're fluid intestines. There's no cough reflex under. They are typically anterior to the testicle. Indirect span is they don't transaminases on. There is a positive reflects the cough Proscar and then the it the swelling bulges. So when you have this, a typical handy thing when you like and hold it and you lost the confident boulders. Barca Steel. It's described as a bag of weapons appearance, and it's typically posterior to the testicle and Japan penniform plexus. Essentially just becoming horrible on spermatocele is a nodular filled subsist above the testicle, Um, my best, So it's rather they'll ask you much about this, But I would I would know these because they come up in past minutes. All What does it mean? SPS? What does it mean in SPS when they say you can't get to puff the swelling? Sorry. What's what's SPS? Sorry. It's a cheap such any. Oh, and when you cannot get Ah, but when you cannot get above the swelling Oh, yeah. Uh, it's a hernia of us. The hydrocele. Yeah, I'm honestly, that's a complete, complete mess. No, but only a really, really experience urologist would do that. I think typically because the hernia is the bowel going into the testicle. You can't get above what you can get above the hydrocele. But the more important thing is the higher dose. You will transilluminate that I wouldn't, um, in real life, that's not really a thing, right? Getting above it, But yeah, you caught. You can't like. So with the hernia, you can't get above it. You can't because it's actually coming from the bowel from underneath because it goes up into the abdomen. Yeah, that right there. I don't really know that, but it's, uh Yeah, you can't, like, feel it distinctly. You can only feel like three sides of it. Some cheese. That's what I imagine that means, But not really right. Castaic your pain Free main diagnoses to consider one is super imagine, see, tortured, Very good. And then there's Yeah, ST I was another one. What's the Yeah, after dinner rolls. And then there's one more which I'll mention that you're on, which is really, really bad. But I've actually seen about four or five times. And, um, nose and not for years, four years would typically present with pain. But I'll come onto it right. So tortured is Yeah, you're right. Now I doubted. Yeah. So your actual Manzie do not mess this because if you do, you will. That suit, um, it's sudden, severe onset of testicular. Well, so there's no promise. Terrible reflux. The testicle attempt to be lying transversus on the label leave at all or left in the testicle. Um, your management's is an immediate surgical fixation off both testes. This important because you, um, as I explained here in the doctor and you've got a what's called a bell clapper, there's for malformation of the testicle. So the test is adhered properly, So if one has tortured. The other one has as higher risk of tortillas. Well, if you're not sure if it's torture in taking theater immediately that just just treat that as a torsion. And if there's nothing wrong, you can just close up and there's nothing. No 100. So Cremins start reflex is something that I've never seen actually happened. But in somebody supposedly when you rub the in, if I the testicle kind of shrivels up essentially like kind of rises up a bit, the more important thing is the lifting on the lifting and know release. If that's does. If that is the thing that neck and pestle corners no relief, then that is probably a torsion, but epididymo-orchitis so respective. You need expression on history of, um, so sex. Ah, it's a gradual onset of unilateral scrotal pain, swelling fever on examination. Pascu that's probably discharge. Hot air thumb a tous. The amount of black is IV antibiotics. It tend to be. I think it's really heavy antibiotic like cataracts. Is that all right? That's pretty much everything. You're that then. So someone's mentioned for you is I can bring him talk about for years, but I haven't even seen the ones, and it's absolutely making. It's essentially in exercise and fasciitis off the Penis, and it is absolutely vial. But if it makes you feel better, Harvey Weinstein have it in the last six months on right, vascular Now I don't know. Yes, come in certain reflexes absence in torsion. But I've never seen it listed anyway, so I wouldn't really the pendulum outs when you're making a diagnosis, however enough. See, if someone has been asking about, I would just say, Yeah, that's a commissary reflex right after dinner. More practice. Okay, Um, typically, it's penile discharge. Hot our therapist testicle. Sort of being generally unwell. So there's two kinds of people who get this. It's either young people. You are, you know, having very, um, so sex. Or it's old people who have untreated UTI That pencil you saw the this little the clientele of this is actually Is that all right? Who? Right mask? So pretty much all vascular conditions. Yeah, come in here. And gonorrhea. But all the two that precipitate pressure alternate ease A. So the main underlying non surgical condition back leads to answers so PhD kind of neat. So also, switch that need to Gangrene. Please just give here. So personal jerseys often presents asymptomatically. The worst that will present with is interferon quadication What? Your risk factor is a P a d. Most common one? Yeah, striking. Everything will give you p a D or you'll die fast investigations. What? Your goals. Some investigation for P A. D, maybe I. And if it's less no 0.9, it is p a d. That's the diagnosis. So your management is antiplatelets. So you start them on aspirin clopidogrel. You can give him heparin. It gets severe enough to need er and revascularization if again it is required. Um critical mst mia. The typical presentation of it. There's the sixteens. Can anyone tell me what? The sixties of critical limits? Give me your arm. This came up in my finals. Parasthesia pain, paralysis, pulseless, holla, perishing. We called very good for whatever that is. I think I tried Peak. You know, stamina is technically the right word for it, but parachuting called. Yeah. Say someone presents with this. It is critical. A misdemeanor. And you need to send to a vascular surgeon immediately on vascular surgeons. Uh, if this happened in a very old person. This this could be palatable essentially, which I don't really understand. It's perfectly simple operation to do, but possible seconds are quite twitching. Doing are very old people. The investigation for is an urgent Doppler bit often it's a clinical diagnosis. The doctor is kind of used to find the exact artery that's clear. Did so, yeah, you need surgeon massacre in but on the master or is a shin. This is one way. If you're not in a tax recenter, you'll put them in a blue, light them over to about the unit, right? Black ulcers. Venous else is these are irregular. These are not that deep, but they look horrible on that kind of surrounded by sort of pussy yellow stuff. Then arterial tend to be a lot more uniform. They have kind of a little they usually above the ankle. Yeah, um, it doesn't have a lift on. They're like others that they're kind of more uniform. The injuries Macey's is on. When I did genocide, and finally finally did they have a bit of a thing about being up to a different, um, venous and then also you also. And that's kind of how you do? This is very much on the parents, right? Case? Right. You are partying and any 55 year old man known smoker presents General is a copay looking a bit crap Norine paid. Which generally worse Sweating, Feeding a big crop Looking great. What are you worried about? What's the What's the main down I seen? You don't want to mess. Yeah, Triple A So what investigation would you do here? Yeah, and ultrasounds. You could do a bedside, Upson what is actually called when they do of ultrasounds in any form this fast. Can you ever get better? Yeah. You do it fast. Um, comes a press it down. Disease blah, blah, blah. I come in the wilds after. How did you get also them, right. I'll go back to this on different gels. It's on appearance. So arterial also tends a heart $10 like this kind of lip. If that makes sense, it's like a couple of millimeters raised above the actual wind. Whereas venous ulcers do not being It's also the rule. So painful and alternate also tend to be painless. Um, and also also Also, it's time to be regular, like a circle where as venous ulcers are like, you know, John Green hard. Is that all right? Cool. Yeah. Triple A. And what's the threshold for a triple A repair? You know, like the size wise? Well, 5.5. Yeah, it's greater than five or five. Big surprise. If it's greater than 3.5 all scarred big guts, they'll screamed. Um, yeah. The other thing is, well, is the grace. If it graze regularly, then it gets and no repaired or your back symptomatic with it. If it if it ruptures, know die. Like if you if it ruptures that in any, you'll have to open them up before they even get better. I had to do that once we I was on nights on. Someone had a reputable A and they opened up in any I let you just hold the aorta. My doctor, about 20. So there's two methods of preparing and play. Um, so it does an open repair on standing like this or does right even or evil. Sorry. Where they're going through the family history, and they just put in a stent. You know what I just said? Yeah. So the point I'm basically getting that whole was, um you're you're throwing the chapter, your local tax very special center that same miles away. He arrests. So what's happening is probably passed. What do you need to do? You know, your last one. Remember, you're not going to go and open them up. Phone double to double, see on start compressions. Essentially, don't you? While in your finals, they might say. Or what is the definitive management you don't need to know? In real life in real life, usually remember a B, C D right? Other basic sagittal things. A swell and I won't spend too long. This would I know. I'm very brown and neck lumps. There's a really good past, my infection on this again. It's that always the one or two questions on past that final Sorry about that problems. For some reason, um, if if the cyst is on the standard cleidomastoideus muscle, it's a bronchial test. Um, if the lump moves forward on up on tongue protrusion, it's a thyroglossal cyst. And if the lump moves moves on, swallowing like bumps and swallowing is a thyroid cyst. But there's, as I said, there's a good actually a good thing on past Medford. No grass pathology. Um, I don't think you guys need to really know this, But again, there's a really department section on most smooth robbery eight and somatic mobile breast lump is like their fibroadenoma. Uh um, a lump. That's painful. Mobile robbery on the present cyclically with menstruation is typically a fibrocystic as breast. Thanks. A regular, broader mass. So followed by a trauma is a fat necrosis, Uh, bloody nipple discharge or a lump is a popular, um, painful. Hot swelling, rapidly increased in size with fever is an abscess. Gradual enlargement with the hard Arabian, the breast lump. That's when you worry about cancer. So, like the invasive carcinoma, this one we got in our final so last with this new collateral axman symptoms of the breast. What's the diagnosis? Yeah, projects disease of the breast. Yeah, which is, uh, normal is, um right. Ah, I don't have time to the spits. I'll be sitting right on this one. So, on a 45 year old Blake, he weighs about 350 kg. Presented with PR bleeding. We fixed it, but as an inpatient became really, really accept it. And we couldn't find the course for it. Hey, um, you since admission, he kind of He's not been cleaned properly because the shades couldn't even properly. He kind of being soiling themselves and the bad we host them that the proper head start damnation revealed several fouls. Found lesions in the medium. What are you worrying about? Am best is like a big surgery, Manzie. So I'm not really a pressure sore. Um, Neck flash. Yeah. So 40 is bang green. Is this specifically in the Penis and scrape him so perineum is not fresh? Yep. So they're not fresh. CT would be your investigation and your management is on them And see the bright men's. Um, these are pretty horrifying, but this happened in real life, and this is quite with it. Oh, yeah. Well, they did from on then. This is the other one. I had a six year old boy got spare, tackled, you know, read the game because the syrup, double pain and bruising, we kind of set him home with some safety. Nothing. But he came back jaundiced and shocked. And what you got? What? What are you kind of worried about here? So any ideas? Yeah. Yeah, it was a Spanish rupture. He had a great four spleen laceration. So this was a CT you to see the big glass aeration and spring my box to Yet you couldn't really different A. A liver injury. It could cause this pain is more fragile. It's most likely going to be a spleen. Then the liver liver is, like, pretty hard to lacerate. To be honest, you just see the them right on. This is quite using slide. I'm not really gonna go through this too much, but postop complications. Fever is the most common one. It's in the first couple of days. It tends to be sort of a specific drug infection, acute transfusion reactions or on a plexus, a little bit longer. 3 to 5. Thinking pneumonia. Think wound infection. Think possibly a collection. A zoo? Well, on great and five days, that's when you kind of have to go for your surgery Complications, like an estimate. Listen to break down the fistula, possibly DVT's. Um yeah, well, so now I'm not gonna get this better. But my mama last thing I will mention this because this was, um I ask you today is the station on taking hand Davis Onda hum neighbors, typically, especially the nurses give you them are shit. This is what I got handed over on my first ever shift as a doctor when I was on the mic shift. That was about friend and somewhere, Doc. My lady scoring a five. So what kind of things are you get on the end of the phone? Going to ask? Yeah. What is going for here? Like, yeah. Keegan. Genuinely. I had no idea who asked him. Do you mind speaking, t Here's the patients. Yeah. In what? Yeah, exactly. So in your finals, they will do a They will do a consultation. And, you know, say all this person's got a temperature on, um, this message got a temperature like what's what's this or what's the matter? And can you come and see them basically in your in your asking? Get as much detail off, um, as you can on but basically come up with a very, very, very basic management plan with the dye You giving you the hamburger? Basically shaving. But you know, you doing, um, the printers? I know I'm being I'm being decision because that, um but that's kind of your. That's kind of a good way to do it. Then finish on. David Lee is your best friend. He will get you through finals. He will do presentations before ah before Final Start. If he tells you something on those, that is the answer because he writes exams past Madison's. That's awesome. I would use it. Finals are your easiest exams in at Embry. Without a doubt, there's nothing ridiculously complicated in there, and you'll be fine. All right. Anyone have any final satchel questions? No know keep, you know, work. Well, they have asked you for the specific up and then take you, uh, also pleased too much a box? A the reason day. This is because I need to do some surgical teaching for such cool training on. They wouldn't really ask you for, uh, say and AP they wouldn't really also for a specific presentation and or specific surgery. What they might ask is, and I don't really need to be honest, I doubt where last use of expensive think. I think that was more I think about more and for interest, and also for, um, like I passed, he found it very helpful. I thought was really annoying when I got on surgery on my first ever sold shipped on, which is nights on but something to do with this person out of right hand. Me and I didn't know what it meant. So the only reason I put it in is I think it I think it's it. It just helps. You kind of visualize what's going on. And then when you visualize what's going on, you can then kind of think right. So what could be happening inside the abdomen? But I need to be worried about is that kind of makes sense to I'll go back to that slide. It was it. Was it the breast slide you wanted? Sorry. Yeah, I put the back. They go again. There's a really good one on Asmat on this, which is really, really helpful. Okay. Uh, thank you, George. Um, I think what we'll do is we'll have a break if everyone could just take a minute to fill the feedback. We're running a little bit behind, but we've got a break Share Georgia bit later. So we should be up to make up time, I think. Yes, it if we reconvene in woman it's time just so ever and gets a chance of another attack. Um, care. Drink water. Right. So about my heart, just go back some nights, right? Um, everyone's fine. So, neurology? Uh, yeah. I got rigid Wilson at the last minute. So it's like, I hate neurology. I think it's especially that, Yeah, I hate it. It's really, really, really complicated. It's very heavily subspecialized so kind of basis. Very, very basically, on f y one knowledge and finals where I've seen uncle for medical teams. Uncle search for teams on the So basically, you'll need, um I don't actually have been year old, just not my hospital. So everything I do newer wise, I have to find middle spur. Hi. Oh, right. So this is kind of what I'm going to base. It will, um, strikes, pleads headaches, neurodegenerative disorders, seizures on be infection. Right. And again, yet, they don't make it a week and interactive. I'm please interrupt me whenever it should go. Any questions? I will try to answer them big. And that was the neurology is not much on this point. I'll do my best, right? Okay, So one. So you're a doctor in any you're on after your Do you call this a 55 year old? He's being brought in by ambulance. Sudden onset, not sided. Weakness reports that it hasn't resolved. Feels funny. Oh, God. It's a sheep chief of funny, almost as if it's a drunk. Feels that speech is a bit swollen. A bit odd. Well, as an f one, what are your priorities of this person? So this is I cannot be very much an authorization thing. So you get given best as an authorization. What do you absolutely need to do? Yeah. So 80 ct had examined? Yes. On most important me. I'm going to come on, examine CT head, Bloods escalates. Yeah, Jay. Very good on so but this one, you would he did a full neuro exam, CT head, and then you'd find the strength team. So this is the CT that comes by on examination to our five weakness and the upper and lower limbs on the left side. Positive pronated drift. Reduce the nasal colds on, smiling on the right side of the face. What's the diagnosis? No, it's not the stuff back. Say yes, Taki or attacks. Very good. So okay. Say that's he's a little bit confusion here. Black on the CD is not fluid in the CT head whites is blood on a CT head CT A black is an induct, not a hemorrhage. Emily is all right. Not any No idea. You know, radiologists and fine. So strokes. It's ah clinical syndrome off rapidly developing clinical off global disturbance. Cerebral dysfunction last more than 24 hours or lead to death with no apparent cause other than Boston origin. So those are the oxygen classification of strokes, which is now kind of widely accepted. So you have tax or total anterior circulation. It's a lot three off unilateral hemiparesis having sensory defect on a homonymous heavy in a p A. Sorry, I am the higher commented dysfunction. So this typically is speech speech is normally the one that's gone on they normally present with expressive dysplasia. So they know where they won't say that they can't get it out. Then you can have a packs, which is two off the question of symptoms. Um, on. Then you have a Latina strike or a lax, which is one off, uh, the above on pox or a posterior second Asian freak, which is a cerebella. Straight, so visual symptoms, brains them symptoms, loss of consciousness for a pap smear. Pox is a much more difficult one to diagnose. Had the emotional one to remember is talk stays. Basically, everything is wrong. Well, yeah. Um, I've messed up it. I should have put him on with him. You know, get in the history of number completely message. So know, technically the parks. But it's a Yeah, I'm just, um Yeah, they actually their sister was a pocket bit. I should I remember. But I mean of urine. So it would have been attacked. Yeah, they're just the important, like, away. Sorry. Yeah, I missed that one. Anyway, the immediate straight management, um, gyn sees he had business up within one hour presentation. You know, it's the specter straight you ignore. I would actually typically ct them before even examining them. And you need a surgeon image by strict team. If there's no bleeding and it presents on the 4.5 hours, you could be trouble. Isis, what drug do you get for? For molasses? Yep. What are the main contraindications off? Formalizes. Therefore active bleeding yet? Breakage. Yeah. Previous straight. Well, that I don't think I have less of that. But leaving this order, I'm going to take that yet. They all anticoagulated high BP. Yeah, and active cancer is another one. Anyone who's been stumbling really place. It needs to stay in Hostel, be observed for 23 days. Why, Yeah, so it's It's Yeah, hemorrhagic transformation. Yeah, So that's quite a high risk that someone, if someone post on the license has a sudden drop in GCS or a sudden thunder clap headache, that would be way. And if they aren't better? Enbrel. Isis. He gave 300 mg of aspirin stops lifelong clopidogrel on this stuff in. You'd admit them salt and busier reduce. Um, you then do a carotid ultrasound. What are you looking for? Yeah, so nose is I don't know what the threshold is. I would imagine it's 50 but it's It will be different, depending on what trust you work in Onda Berries. Signature. It's directs, the nicest you would before the mom to a basketball team. Yeah, that's what all you all you need to do the stroke and then everything else with the stroke is managing complications on so physio and salt induced, expensive extraction. Aires patient wakes up with deficits, so presents a hostile with them. One hour can be traumatizing. No, you can't. Trumbull eyes. Um, because, um, because you've no idea when the deficit started so that could have started at 10. PM when they went to bed. Yes. Say, if they went to bed, Well, I woke up. Um, well, you would. They would not be incumbent for the nebulizer because they're so much I weigh the benefits you couldn't do. We do. A throwback community could see exactly what it from this one. It's and you would typically not. What have they just had a short nap? They're asking The real question is a great, uh, if they just had a short not I get it the pens of No, she doesn't. If they really slept for, like, 20 minutes and obviously, yes, they could, Then after, um, well, Isis, but typically the temperature transmitters to go to bed at midnight in the wake of 80 with doctor said you didn't have any time in the last day hours. Uh, patient presents of all signs of a stroke. But CT had changed nothing. Your treatments of the strike. Still what's emerging? Would you then go to Yeah. So you admit to be cheap interest rate. But then you do an MRI. Yeah. Cerebellar sports slash T I A This is horrible. Best was actually my osteo station. Eso What symptoms did you get it, sort. Sorry about that stuff. Danish. Yeah, very good. Say what? What? What? What are the components of Danish? What's the eye? You got the first three? Yeah. And then here's an abnormality on the eight. That actually the s Yeah. Flood staccato speech. Yeah. Get this side of the kidney cysts. Um, attacks the slackness. Intention, tremor, scanning dysarthria slash slash speech. The ocean test. Yeah, and yeah. So if you're expecting a cerebellar thing, you got to make sure you do four of those. Okay, G A is, uh, patients who presents sudden onset broken urological deficits that have completely resolved within 24 hours and cannot be explained by any of the condition management for CIA. I don't wanna have the gas slash not a guest. So anyone want to tell me? Okay. Yeah. Say Julian's platelet therapy on a diet clinic with the stuff in doesn't actually need to be within 24 hours. Um, so the definition See, I should I don't know if it has, so I have not be. I don't think it has bits. Um, it's now tissue based. It might be for your exams, but it isn't over your life. And I have not seen a straight line. So to do that and they don't need to see the head. Yeah, I'm gonna leave that one with you guys. Because a time where ast far as I have clinically seen a tia is just woman's lost 20 less than 24 hours with no ct head issues. I'm I think, Ah, I'll leave that with you guys. Um so nice. Okay. And is what's the most important thing to advise someone about? He was having a TIA or a stroke. Yeah. So something smoking. But driving is the medicine ball. When driving is the one that they will be looking for a on asking if someone 70 a it kind of ta, they can't drive from them. If they had a strike, they can't drive for several months. Um, working. So if they work in HDV vehicles, if they work in and that's all anything. Rebels big. Get your driving machinery. They They can't do it. Don't need a character, especially for older. They've been left with the deficit. They will need carrot input. Most people who gave you this will need rehab. And, most importantly, stop smoking. Anyone is active. Swelling around 60% chance of recurrence. Strike? Um, yeah, that's like, Okay, that's much everything with strokes. Sorry. I don't really know the new definition of T I A. This is one that I would ask the cloth about. Um, I also frankly, don't think that anyone is going particularly care about the minutiae over diaper off. What constituted strength? Pretty. I I don't think it's that important. All right. Right. Blading. So, case one is, uh, case one is a patient with sudden onset from the club, huh? Doke? Which work them up in the night. That is the CT. Yep. Very good. It's a summer, right? Um se can you see the we white line? Just there about is the bleed in that. Then next one, patients press presents feeling and priestly confused. Lots of falls at home. But the CT, what's the diagnosis? Very good. Subdural What ti groups of people are particularly high risk of subdural that sort to collections. Alcoholics on the elderly. Very good. Yeah. And then number three patient presents asking the source of my club being hit on the head to the baseball. But but is the city doctors and extradural Very good. Yeah. Oh, I said that Also shaking baby center and give you some gel spirit. They go actually, your hematoma. So your management's for Subbarao on, uh, a B C D E. Treat anything, treat any other abnormalities essentially optimized for immediate neurosurgery. They'll need nimodipine 60 mg every four hours. Where if they thought seizing, you could give them IV anti epileptics. Um, if the CT had shows nothing and you're respecting a suburb back, we'll do a lumbar puncture. Love a bunch of will be positive xanthochromia, um, boxes enough for a diagnosis of a suburb. Subdural ABCDE on treats. Any other abnormalities monitor prophylactic of the epileptics If they have a midline shift and then actually really mature under management of ritual bath, great injury units and everyone needs the have accurate that I don't know why you're getting about being on some Iraq, and it is what I was told to do my neurosurgeon on the weekend when I was working my hands and move around, but that I don't think it's a question of a very good preventive, is a spot. It's what I had a very angry neurosurgeon. Shout at me are saying why I haven't started them on the bubble mean because it wasn't in our guideline, right? So GCS is It's very important, Teo and very important to know how to accurately do. Geez, yes is and it is a lot harder than it looks to do it in practice. So I've got a couple of scenarios off. JCs is and you guys can tell me what's scoring You think they are so 61 year old lady that makes it after a car accident. She's not warranted Time place, but it's responsive. I mean, there was a minute spontaneously. Yeah, he's got 14 on but would also say if you ever all right and I'll see and so one also you to do a GCS I would also break it down. Uh, 18 year old girl, heavy a period overdose, making in conference with noises flexes away from pain, but not opening eyes. Oh, yeah. Jeezy s Do you go the most optimistic? Yeah, if it's between two. I would just when I was having a someone, I would say it's between 13 and 14, but fluctuates. I'm not. I think it's reasonable, right? Say, flexing away from pain. Um, incomprehensible noises. I'm not opening eyes. And we also have a guess on this one. Yeah, it's six. Exactly. Um, this is a really silly point, but I'll give you what, actually, wires of this. Make sure they can hear you. I was working in 80 at number with Janet Skinner, and she was trying to do a GCS, and someone was shouting at them and she didn't realize they had a rods in so they couldn't hear. So make sure they're not wearing happens. Make sure if they have hearing aids they have eaten. So you can actually make an accurate GCS right? Is that all right for bleeds? Don't have any other questions. Right hand x case. One two week history of a headache describe so tight band bilateral pain. It's almost like wearing a hat on his head of pain. It's not debilitating diagnosis. Yeah, touch of like and no investigation required. Simple allergy. Easier reassurance on West Wing advice. Basically limit. See sudden onset of moderates. A unilateral help. Pain pulsating, debilitating. Proceeded with flashing lights and mining bolts. Yes, Mother. Very good. Managements or migraine. And your moods have the gas or not? I guess so. This one, I think, is gonna be one of those. That difference between trust. Your first line would just be basic analgesia. If they have returns migraines, then I think you can give him trip tons. I don't. It will be very weird to just get someone tripped on because they have migraines every six months. That never that would never happen because they have horrible side effects on prophylactically. Propanolol on two permits if they've hired migrants for greater than two months regularly. Uh, very obvious question. What pill can they not have? Yes, called Have the same sleep. Very good. Right. Next case. Three. Patient has any dairy agitated. Severe pain in the head, localized in the left eye. Watery on diagnosis. Cross addict. Yeah, very good. What's your management for class? The headache. What? Yeah, height. I want you on IV energy. Easier, So just give them heart like low depend relief. And I fell. And you and your partner tasteful, uh, severe had a loss of vision and your pain. Uh, on further history, they've been sore hips and shoulders. Months? Yeah. There we go. Dinosaurs GC a, uh, investigation. They need a new Yes, all. I don't need a temporal artery biopsy. Yeah. Um, and then management for GC A. Yeah. What? Those? You start Mom. Yeah, 60 mg pride on you. Then put them on a winning races on this wing Regime typically takes about two years, right? Having red flags. Do you guys want to give me some red flags of headaches? Oh, honestly, I have absolutely no idea. If you give me the credit they lost. And if they have vision loss? Yep. So his malignancy vomiting, fever, sudden onset, severe deficits, personality change, produce consciousness, trauma, bread, painful eyes. Stangel change in pattern sticks. Yeah, graze. I see the signs. Oh, yeah, right. Painful. I This is actually writing board one. What if someone come in with severe severe, severe headache and red painful? Like what I'm actually thinking. Yeah, but you angle closure. Very good. Yeah. All right. Basically comes in terms of headache, red flags, bids. It's just the board to kind of recognize which ones you're looking for, right? Is that all right? Your headaches and all right. Any other questions? No. So it wouldn't be anterior uveitis it. Would you be more in about a Q and a pleasure? That's the one that's like kind of the super Super Super Emergency and Jerry you get is the fancy as well, but it doesn't need to be fixed. But within the next hour, my SS see an ophthalmologist 70. Well, right. Any other headache? Questions or concerns? Nearly degenerative disorders say case one 69 year old man since one year. History off. So slowness, loss of that 70 handwriting's become worse, Wife feels is less expressing his voice Got a bit softer, sudden trauma. What you kind of thinking, Yeah, so sort of Parkinson's type thing. Parkinson's symptoms are the acronym is trapped. And then I want to give me the the T eight t r e p. Say What is the tremor described as What is the rigidity? Describe that Yeah, Hill Rolling tremor. So they just do this with their hands. That's and what's the virginity like so not lead pipe. It's colloquial rigidity, but right, so it's like they can go like that. Essentially cover Jersey A is bacon easier or Brady can easier on P is only kind of late on stage is at, uh, postural instability. So management for Parkinson's specialize, But these the most important drugs you kind of need to know, right? So this actually is kind of the one most of the start from is a monologue. Today's beater inhibits Uh uh, probably carbidopa is a dope decarboxylase inhibitor on l dopa, which is the the the main one, which is the precursor for when you're taking issue, someone that you think of Parkinson's disease. What do you want to make sure that you're not messing. So that's some things that can kind of predisposed symptoms like this, that you want to make sure that you have ruled out before thinking someone has Parkinson. So your urologist doesn't matter for you. So a normal person with hydrocephalus Yeah, I mean, you'd see if he had them when you see it on the CT. Had medications yet to this tomb, two groups of medications that you need to be careful and one is I'm psychotics. Very good. And then another one is metoclopramide. Yeah, but they're on long term that they can go back and cause parkinsonism. I don't know why anyone would be a long term. That's what might It's completely useless and point this drug. There we go. Right case, too. So it's a six year old bloat presents with the right foot drop, which is developed gradually over the last year, involved more proximal areas last two months. Associate Marshall twitching, cramps, bilateral weakness more severe on the outside. Spaciticity actually so Ah, whole load of neurological operated Your insulin is the arm. Very nice. What kind of things are you thinking? Yeah, Yeah. Sorry. Sorry. Right? Mnd Yeah. Okay, sorry. Mostly disease on a less, which is the most common presentation. The most common subset of military unit disease to make the disease is an umbrella of traditions, but ah uh, a a less or a come before they actually stands for But that's the most common one. So it's a gradual onset of operation. But the weakness, stiffness, core balance, coordination, spasms, spasticity. But the foot drop is often the first sign on bag. Know sees of em and er often missed because foot drop is also a presentation of a specific nerve palsy. So people just give them the diamond specific more of palsy on. Um, they didn't, uh and then they get sent in. But then they come back with, like, an aspiration because they got a M and P. They talk to you actually and hyper or fax it. So your management for this, it's that it's not new. Basically, just told this isn't curable. It's almost not even treatable on it. Will probably tell them you can give him raise old, but it does it that very mildly helps that deteriorations is open through aspiration. Or if you just respiratory shutdown because they die from isn't innovated. I think the basic points cigarette from M and D is not. You're very unlikely that you got this and then all scape on if it's a short, best answer question on, but it's a lot of very upper motor neuron type symptoms, and you don't really sure what that is. You want to make sure you hold this out basically. Okay. Case three 70 year old woman brought into GP by her Children because she's become more forgetful issues paying bills in the penalty and enjoyed cooking. Recent received allergy notes is when you turn, the companies are comparable. Deal lost 3.5 kg on last night, left water running in the bathtub and it flew to the house. Thank you, Princess. What is the diagnosis is you're looking at here. Dementia? Yeah, so it'll be hard to kind of diagnoses outside mints when you're not on outside of specialist. Basically bit dementia is kind of were thinking. Yep. Are they the point under to try and get out here? Is that you? As an F one ordered enough to even a range will not be able to diagnose outsiders in of itself. You will be able to diagnose it. You'll be able to suggest someone has the lunger. We will never be able to die is it's because you're not a specialist. Presentation wise, progressive memory loss, disorientation, apathy decline in commented physical activity. So it's not a clinical diagnosis. It needs specialist diagnosis. What you feel that everything else basically, manager is largely supported. You can get cholinesterase inhibitors, but it kind of depends on the specific type of dementia. So outsiders is the most common type, and it's a slow onset. It's characterized by what's in the brain. Um, beats amyloid and tau tangles. Yeah, very good. You don't know what they are, but that neurological buzzwords toe mention. No, Everybody is typically a behavior type. Um, um is a behavior type of dementia. Boston of dementia is very rapid onset on Parkinson's. Dementia is another. So some sort of dementia where they got so Parkinson's and dementia at the same time. The point. The point of getting on the renters out. Somebody you know about Taltz angles and be traveling, but you won't need to know like too much about the lunch. It's just a very important thing that lots of your patients will have on top of other things on. It's starting to bear in mind because they're much higher risk of things. Like in asked in the near Eames in hospital behavior changes on refusing meds, right, New confusion, screen master. Still, this is quite important one and one that because I'm on geriatrics in a minute, this is one that you'll have to be quite a lot bloods, so full blood count. What in a full block. How would you be looking for? For an acute confusion. What would you eat like looking for? Yeah, high white cell for infection. Using these. What would you be thinking? What you're looking for? Yeah. Okay. I and lt's iron studies thyroid. All of these town precipitate a delirium or a new confusion in hospital urine dip for an infection. Chest X ray. If your chest infection, yet you send it off in embassy CT heads. When do you say when you say, What do you say when they ask why you're taking month taking lots of bloods, taking allergies? Uh, very good question, I guess. I guess that it remains the main thing you could be worried about a couple of the but someone's got acute confusion. Um, you'd be worried about a couple of the especially their abuser. Or, you know, you can take an operation that you want to see. If that possibly Joan dust on the as a baseline increased often, that's something important, one that you know when you get many and, uh, Wilson's disease to be a really want to say for a new confusion and else. But yeah, allergies are pretty standard. Let's just they wouldn't be to him some of that. So your hyperthyroidism pants if it confusion on B 12 deficiency, and I and so these candles accords confusion, you probably should Yeah. Uh, well, you probably would need I don't Counselors as well, actually counselling magnesium basically just like full blown the living daylights out of and see if you can find anything wrong on try and fix it. Calcium doesn't tend to prolific too much confusion. To be honest, that's That's more so as a result of the confusion, they'll have falls on their know get so hypoglycemic guns, like have pathological fractures, right? And the only balance lower jobs cause constipation on deuce oil and take can also precipitate delirium. I'm you confusions in a young person in new confusion. What other kind of test would you want to think of? So they say all of the above would be the kind of older people in geriatrics, So yeah, bm top screen. Yeah, that's reasonable. What else did you try to be thinking? Yeah, yeah, I I would do. And I'll be in a younger person who's come in acutely confused. No. Well, the immediate reaction isn't something psychiatric, but then you need to do anything else on. So I would help. Even they're very good. Okay, Next 25 year old recurrent slurring of speech that worsens let you continue to talk. Just trouble snoring tier rates. And she continues to eat. This is double vision. Gets worse. Starting reading or watching TV, she pulls. I had this had being hard to hold up a citizen progressively deteriorating the last six months. What do you worried about? Yes, Very good. Myasthenia gravis. So it's a crying. Also mean disorder or a presynaptic membrane. That severe muscle function at school muscles Eso presents with progressive fatigue. I stuff dysphagia and dysarthria sometimes impossible. Lead weakness, investigations. Well, that's what you gonna do for mg. Okay? Yeah. What to you antibodies. You looking for so anti? That will not start yet. And then there's one more musk. Very good. Well, obviously ct them as well, but these are the ones that you want to look for for myasthenia gravis. Musk in a chr. The amount of Brits steroids immunosuppression because this is empty. And also, I mean conditions. So you just trying to pressure immune system part Cigna mean is kind of the a c R. Inhibit. So that's quite useful. One as well, right? They're all the and once they have not mentioned I'm not mentioned. Gambari have not mentions most will cirrhosis. And I think we should have mentioned multiple sources bits and that I will leave you to go and research yourself. Right? Seizures. That's a cuz. Histories the history of See that you need to take extra before, during and after. This is a really cool classic ski scenario. Is someone's come in with Just had a seizure. I want you to take a history you need to do a collateral collaterals. Very important because someone see if someone's got a video of the seizure. What initial investigations would you do for a possible seizure? Yeah. Bm What else? Bloods. Possibly a CT yet? Talks. Yeah, he's itchy medicine Born Yeah. BM bloods, drugs, alcohol, CT head. Yeah, pretty much the full work. It's right on. Obviously, the first time someone sees is you cannot bag nose. And with epilepsy, you just need to try and find a cause for the seizure. When if you call it clinicals of the seizure, you then meets a preferred the moment you're in your oldest right epilepsy. It's recurrent seizures, which is made by a dog nations made by a specialist for the tea subcategories Generalized, which so both sides of the brain are essentially fried. Immediate loss of consciousness. Chronic tonic tonic. Clonic eight on it. My clinical absence. Seizures. Or you can have you can have focals, Egypt. Focal Epilepsy, which is a specific area of the brain that is not working properly, and you have specific types of seizures. These come then general develop into generalized seizures, but they will typically be very, very specific types of seizures investigations. So an E E g is sort of the gold Sunday after laps the investigation. But that's kind of only really done. And interpretive Last fascia list MRI. You would definitely do an emery if it's a new epilepsy in adulthood or it's under two years old. Focal epilepsy. Uh, why would you do? And I'm all right if it's a focal epilepsy? Yeah, you You want to rule out Shima that very good on See, just continue despite first line treatment. So we this is pretty much impossible. Do you get them are eventually. But if you give them sort of operate on, they continue to seize your end. Want to see why? Because this is not responding management. There are loads of anti epileptics. You don't really need to make too much about them. Valproic is still the best anti epileptic. However you can, you know, give operate to Yeah, very good. So if you are very is now one absolute contraindication of pregnancy, it used to be all we'll try and see how we can make it work. But now it is an absolute going to vacation. That's very good. Psychogenic nonepileptic seizures. This is the more common causes. Seizures. It's kind of very subtle things in the history that would make you think it's ah psychogenic thing. Rub in the natural epileptic seizure. So So I took this, uh, besides from last year, some of these your bit decisions. But the most important one is they're not postictal. If they don't become pay strict well after the seizure and they're pretty well, it is likely a psychogenic seizure, um, against. So these the ones you always want to ask. You always want to investigate in a seizure, tongue bites and incontinence. And if they were postictal afterwards, that would very much guide you towards an epileptic type procedure, a raise productive and a raise lactate or also seen our procedure. So you're after post blood. You want to take blood pretty, actually, And see what, that day Right on status. So status is a single seizure lasting more than five minutes or multiple seizures without fully recovering in between? Um, this is a medical emergency. Um, if this will lead to so brain damage, the management is essentially integration IV access. If it last fight longer than five minutes PR, diazepam or IV lorazepam. Repeat once after 15 minutes. If it's after 30 minutes you gave her I D. Or I only found a tear in with after 60 minutes or they're under a factory. States issued intimate them in seven trade sides. You and this would pretty much be a crash course. Um, and if someone was in stuff just because all of these would be way above you and I don't think we can actually administer IV lorazepam or IV Beneteau without, um, I see you're on on the PSA test on site. Yeah. So status is it's want to just kind of be able to spot on desk right immediately and then one last case. You cannot miss whether you are in GP or in any. So you're working And after GP Mother brings a four year old daughter She's not being itself, has vomited ones. She's no eating or drinking and developed a rash from her back. She's got a high temperature on flexion off the neck causes flexion of amusing hips. So yeah, this is meningitis. Pretty much a minute. Medicine, Yeah, say meningococcal septicemia. Yeah, so most common in teenagers, Children on the elderly, fever, nausea, vomiting, malaise rush. And then you got the trio. Next of this photo failure Heartache in a young child you have a bulging from to now signs a shock on you. Also that specific a little signs so can excite Is resistance to the extension of the leg when the hip it's flexed, this is a really common one, and then bread, since keys, which is flexion of the neck, causes hip and knee flexion as well. Course of organisms can be viral, but my cereal managing this is the most common one, and Children, uh, strep pneumonia is the is the most common one in infants and in the elderly, and it's very dangerous. And he may. Influenza is another one as well. Management and I would wait for a healthy for a meningitis if you did get the meningitis to treat you as a meningitis. So in the GP you'd give iron band pan. Um, use in the hospital need to do IV calf track saying, if I would also person to give IV acyclovir same time as well because, um, it could also be encephalitis, and you're not sure if they're immunocompromised. You give amoxicillin as well. Plus that, Smacks said on the house, I'll contact you to get kind of days of separate boxes, and right, so that's how much will have the meningitis. Meningitis just wanted. Cannot mess, the very unlikely for someone to ask you to diagnose and capsulitis. But if someone came in with symptoms off meningitis, I think it's plastic reasonable to give IV antibiotics and IV. Uh, I'll be a cycle there. Oh oh, done. And I can't remember when you get that there is a specific. There is a specific, um, threshold that you would give That's messing for, um, it's on past, Um it was my body Ragged, You remember? Was when you get back to myself. Meningitis? No, no, it it Yeah, it's something that you wouldn't do unless your Reg told you on. Right then. General Neurovascular iced. So neurology constant. All the sections of tend to be written by Richard Night. So that really mean think, I think what? They're actually what they actually presented you with because they will give you lots of lots of red herrings and bullshit. I think exactly what is actually going on before making a plan. Say if if there's lots of lots of other red hands in one. But the main thing is weakness than think you want to rule out stroke. It's a part of the thing for us. Is is the safety factor. So if you're worried about them, I think, um, it's the baseball thing is thinking all you worry about this patient. Basically, yeah, I'm being upset on kind of stress is probably their home. Or is this like what I need to be quite worried about a quite severely from well in urology. I think it is reasonable for everyone to get the standard slower investigations, including the CT head, because you don't know what's going on. Yeah, well, I'll put this out because So my posture station welcome. I'll show you what mine was in a minute. But the individual eight er is there to help you on day will die. Gee, if you completely miles off trucks and, like, say, near near, um, May only that your is Asians are rest of my rich, my liver being horrible. Do you see the hand? Should new policies, I would. It it It is entirely reasonable to, I think, if they are known, Apple actually than know. But yes, it is perfectly reasonable to see the other child. Is that a seizure? Yep. And this is this is my ostentation. This is All I got was a fortune one presents the am you with a short history of dizziness, taking shooting down. So that was what we got. And what do you guys thinking? What would you need? Where would you kind of go with this? Yeah, so exactly best ladies. Things that it could be that could be going on. So I started doing this with a cardiovascular shakes. I called them. I would simply but then the individual a to stop me and went Now you seem to be going down. Cardiovascular gave Teo another specialty. I'm being kind of help me through. And it wasn't cerebella t i and eso, but the point I was going out with this is they're deliberately Vaid because they're gonna try and throw you off. But it will be fine, usually to kind of but cheese a hill to die on. And they will tell you that's the wrong one. So for a cerebral a Tia, you do You try and Alicia elliptical the Danish signs on this. I don't know. Kinesis a taxi is you get to walk the stagnant. So you do. Uh, creatinine is 45 and six, and I Attention pharmacy do so the and the coordination days pointing exam asked Awesome to, like, repeat phrase back to you, um, on a mail. The fusion things. Yeah, yeah, I wouldn't do a heads I would Very much not doing. And she needs out of training. Otherwise, if they've got an undiagnosed cervical spine injury, you can cause quite significant damage. I would not do a hint. I would say you do a hints, but you don't know if they've got any cervical spine issues. I definitely would do it because that's quite a M. It's quite a, uh, dramatic exam. That one. Right. That's very much. Or add the nearest don't have any more questions. Him. Thank you so much. Uh, Georgia was pulled last minute to come. Teach this. So they think it's him. I put the feedback form just in the chopper. Um, would ever and appreciate a break. The full senses. Oh, does it does everyone of just straight in a break. Okay, well, have a 10 minute break. Chelsea. Is that okay with you? Yeah, absolutely. See you guys in 10 minutes. Okay. Oh, my God. Hey, Chelsea. Okay, fantastic. Okay. So hopefully everybody's at back. Um, Hi, everybody. I'm Chelsea. Uh, I'm in a room one. I'm going to be covering the senses lecture today. So, um, um, era was kind enough. Teo, help me with be making this this lecture. So he covered the dermatology. Ophthalmology side won't be here. Today is just me. Um, but I'll do my best to cover his slides. And, um, so your questions okay. All right, So today I'm going to just cover anti ophthalmology. Dermatology is going to be more. It's going to be less of a lecture and Maura of a kind of like going over some questions with some knowledge because I think, um, in terms of the senses questions, you're going to be pretty covered. If you could just select the right questions to know, Um, I'd say some knowledge. Obviously, you need some background knowledge as well, but you would need it as in depth knowledge is per se are like neurology, you know, um, so hopefully just a sense of things today, Um, I'm gonna be going over the questions pretty quickly are apologize beforehand. We'll give you a few seconds to think, but we'll move quickly because there's lots to cover. And I know this time myself Last year I was taking lots of screenshots. If you need me to go back, if any questions or you want me to go back, take a screenshot of something, just let me know. Okay. So for their do, we'll start with ent. Okay, one and so 50 56 year old man presents to eat it with a piece taxes. He's known mechanical heart balls and he takes 2 mg warfarin. Easy. It is most recent in ours, for he's lean forward pinching his nose, but he had to stand a stream of blood from the right nostril. He's been bleeding. Oh, my God. That was a key thing to stop it from my face for 30 minutes prior to arrival. What's your next management? So they asked him to pick his nose correctly for another 15 minutes and we assessed be prescribed vitamin K, pack his nose call ent, see? Use ribbon dressing and packets nose. Take some basic bloods and prescribe trying. Examine acid D from your CT silver nitrate a rock rhino. Establish IV access. Take some basic bloods and prescribed tranexamic acid or a bleep on call. And he, uh, one. Okay, people, most people said, be so the answer is actually Oh, D um, So I think in the first instance, as like, you know what, everyone on the ward What you want to do is, if you're able to do silver nitrate cautery, that's always the first line thing to do. If you can't find silver nitrate or it's not in your emergency department, you can try to find a rapid right? No, um, things Even if you if you find like, Oh, my God, that's about my head. You can definitely still get IV access, take bloods and prescribe trying exam, make acid. So in terms of the vitamin K, there's actually not some hopes or what is a rapid writer. So robbery know is just the name of the specific packing. Um should be told it looks like a tampon. And what you do is remember what they have in loaded. So you, either and it looks like a tampon. And you either inflated using like an empty syringe with just like a row. Or, I seem to remember you can also put some saline in it and then also just expands. So that's what about the Rhino is, and it provides some basic compression. It's different from a ribbon dressing, actually, but just think of it as a tampon temporal. So why? But, um, in case is not the answer is there's actually not really it's not really indicated here to give right. Um, in case I mean this person, you can't stop the work because they probably taken it this morning. There isn't really much of an indication to take them off the warfarin pieces person's heart balls, and it's not really in the management anyway. Um, usually what's kind of helpful is, I would say, Just stick to epistaxis ladder. You find this easy on the on the Internet, and it just provides a pretty good like step two step About what? To do that sound. Okay. Okay. Someone is leading on warfarin. Do you get Procrit? Was that only for serious bleeding? Oh, I'd say, if it's, um I'd say, like obviously, this is a very hypothetical djinn with the patient in front of you. It was major hammered something in their abdomen or something like, Please, port. Quite a bit of blood. I would say I would run it past senior. Maybe. Why not use IV with them in case away? Um, well, because And it's just not it's not a big enough bleed. I suppose it's not indicated here, and it's not usually is basically know what they normally do. Then it's probably You could do that. Yeah. Yeah. So I think those are all really solid reasons why, um but in any case, I'd say just don't be thrown out by I'm seeing Vitamin K is one of the answers. It's usually not really in the kind of management. If it was like a major, like, you know, buckets of blood, I think you'd be a bit different. But in this instance, I don't think they would throw you an answer with vitamin can. Expect you to kind of treat that one dot All right? Okay, I'm sorry. All right. So moving on to question to 27 year old man presents to your GP complaining of stuffy nose, the nasal pain, loss of smell and relentless yellow colored nasal discharge and peeling feverish, that is. And he feels like it's been getting worse in the last 10 days. His cold PCR is negative. What's your initial management? So A analgesia and worsening advice. Be watching weight. See, Charles took his own nasal spray. Um, give some saline irrigation on and just review him in about two days and d give cool marks a club, um or, uh, and give him some allergies and decongestants. Or you referring to the ent Emergency day click to give that a shot, guys. Okay, So most people went with C and so C is a very south solid, um, and disappeared. Solid answer. But the answer is actually D in this case, Um, but you weren't wrong about C, but it's kind of you have to be able to differentiate the different, um, levels off. Uh, like at this stage, it would be indicated to give this. So in terms off what on the Oxford handbook says is that there is usually stage acute sinusitis in terms of this mild, moderate and severe and mild is more. That it's less than five days seems to be improving and you can actually manage conservatively. And the stuff about that with moderate is that it's been kind of in that kind of more gray area that's been over five days. Don't seem like your typical cold. Um, and yes, um, there's more like more symptoms, but you can actually just manage that with the tickets. So So, yes, this is C is actually a very solid answer, usually, but I tell you, what would take you off in this question Stem that he's needing a bit more is that it's been 10 days he's having. He's very symptomatic with nasal pain and he has colored nasal discharge. He's not. It's not getting better. Um, it's been just such a long period of time. You would actually be indicated to give him some antibiotics at this stage. Um, yeah, exactly. So he's just not feeling better at this point. You know, you could give Fluticasone, but is that kind of If it was me as a GP, you know that Fluticasone might help him, but the direction it's heading, he'll probably be back in a few days saying it hasn't gotten better on No, no need to go to Emergency Day clinic. Let's see, Obviously we would not much weight. Fine. Okay. Decongestant. Well, it's just kind of like over the counter stuff like my mental Afrin. You know, just what you can buy, like, sort of fact, that kind of stuff that can help kind of open up the airways a tiny bit. It's nothing. Nothing fancy. Okay, so okay, for the sake of continuity, the same patient arrives back at your GP clinic a few days later with a painful, swollen right that won't open, he says his name's and sometimes have improved a little a little afternoon biotics, but What's your management? So they advise him to finish his antibiotic course, and we'll just reassess that. Be examined I and describe for me beautiful eye drops, See, prescribe second line and bonnets and form Pinnacle Eye drops D performing examination of bloods and said in urgent outpatient referral to ent or examination and bloods call the on call ent and send this man to a me. Oh, shit. Sorry. Pardon my language. Oh, no. Okay, pretend you guys don't see that. What was management of less than 10 days for the last one? To be honest, it's been of, ah, more gray area like I was like pouring over the handful of being like, What is the answer? I see. Yeah, just go from the question stuff because, um, in the middle, you would kind of be they'll be quite obvious in your precious time. It will be more like mine is in like overkill is that this guy is not improving. He's not looking good, probably needs antibiotics, or it's going to be kind of in that kind of gray zone. And I'd say 5 to 7 days, in which case you could manage with just the tickets own at analgesia. And it's not quite past that threshold of, um, saying that you need indicating on antibiotics. You Thank you, Hannah. Cakes about the pool ending. All right, well, so pretend you didn't see me upset. Yeah, you also the answer. Good. Okay, so the answer is he, um So hopefully you've picked up that this is, uh, peri orbital cellulitis on this man. This is an American. Seen this man. Basically, he needs to go to 80. So you need to call it, like, basically do your thing here, but call ahead to ent and refer this that get him to 80. Um, he needs, um, ent and off multiple input. And he probably needs a CT. Um, I'd say don't don't fast around with this one. Um, yeah. Since the I use the kind of the same patients before, as a sort of like four. Used to, like, read last. But it's not completely wrong. Like most of the patients will have a history of some sort of respiratory tract infection or some kind of like Sinus ethmoidal infection. So, yeah, have a low index of suspicion. He's going to want it slow out of hospital. Oh, yeah, that's true. And yeah, no, that's their enough. I'd say Send him to a any Then you take some blood there. But you're right. It would probably take a few hours, and I'll take them and 80. Okay, get a full set. So do you think Okay. All right. So if you sorethroat cases, um, question for so gp refers a 14 year old girl. Ta be complaining. A fever, sore throat, pain, an ability to swallow. Um, you look inside your mouth and you see the falling. What's your management? So a blood IV? Fluids IV, ben pen and dexamethasone be bloods IV fluids or a little guard fan oxymetholone, penicillin and Dex. Um, see, I am been pen destroyed with worsening and vice and give analgesia D you aspirate it? Given anti z a difference, which is like a like a mouthwash or a, um he aspirated and you give So, by the way, on this or that period, orbital cellulitis. Yeah. I don't want to click in case they get the answer again, but that's me typing it. All right, Well done. So the answer is a, um So what? When you hopefully identify it is that this is tonsillitis. Eso just to your usual thing like bloods IVF then and then you give I've been pendant dex. So, um, the key is here is that, um the reason you want to admit this patient is because they're not able to swallow so any time to the patient has, like, like that they're not able to tolerate or Alimta take they have to be admitted on. But why you give them IV fluids and why you could why, like I need, uh, antibiotics is indicated. Um, dexamethasone is there's some evidence that it helps with the pain. And that kind of helps, you know, remove the infection down with things down a lot faster. So you give that as well, just as per your local guidelines. I think it's taste for examples like 6.6 mg. Um, do does that make sense? So if you look inside the bowel, you can clearly see that, um, if the tonsils that unclaimed and you see some exudate um Yep. Anyway, plastic appearance for tonsillitis. Okay, so this is center criteria. They sometimes referred to it. Um, yeah, it's pretty straightforward. You see these things or you don't. But to be honest, if that patient walk into the door like, you'd be pretty sure you don't really need center criteria to know. But for the sake of writing sounds, it's worth knowing, um, higher score. Just mean that's more likely to have structure Caucus. Okay. And another thing that they sometimes like to ask is one of the indications for tonsillectomy s. So it's seven infections and one year, five infections and two consecutive years or three infections and three consecutive years. And there's some other indications that you just don't need to know. But this one is kind of, like, easy to test. Okay, So very similar questions. Um, uh, GI refers a 14 year old girl to any complaining a fever, sore throat and pain and inability to swallow. She's difficulty opening her mouth, but when you do manage to peek in, this is what you see. So what's your management? I think I gave pretty much the same thing. Oh, except for the dex. You know, give it a whack talk in a minute. So, um, pull hasn't come up for me. Anyone want to you offer an answer? Where did this change? Okay, so I think we're just march on. So the answer is, in fact, he it is Quincy. Um do you do so I think the key points here is that hopefully you guys can You were not me about, like, you know, at some point, you know, and I was a bit more confused about, like, tonsillitis and Quincy, So hopefully you can see if you look inside the mouth is pretty different. Um, my goal. Yeah. So, as you can see, like, if you can sort of remember what the tonsillitis looked like. It was clearly the tonsil that were inflamed with exudate. Whereas if you look at a very tonsil or abscess, it's not gonna be the same area. It's just a different appearance. You're gonna have some. You might have some. You pull a shift. Um, there's usually this kind of classic kind of trismus or locked all and they'll have a hot potato. Boys didn't put that in the stomach. So what? I'd like to obvious, but usually they kind of like like I kind of I can't really copy just now, but pretty distinctive, um, in the management for this is actually almost identical to tonsillitis. So you actually do want to give, um I need then pen and you want to give you a Dex or a Dex wasn't here. That's what I do is that s o pen pen and dexamethasone and but apart. Like I'm talking that you want to, um, aspirated it as well. Um, so yes. Wait. Sorry, Roy. Differentiate between which two? Oh, excellent. I'm Canadian. I actually don't. Okay, but are we happy? Are we happy? Okay. Um, yeah. Okay. So aspirate and of aspiration does not work. It's gonna be incision drainage, but that would be like getting to bridge. So moving on. Okay, So a question is, Are you remembering is in his 18 month old daughter who was short of breath and Parex it. The child is restless, drooling and lean forward. How do you manage this child? So a administer I am happen, Afrin being full examination, including over Ferencz. See, gain IV access, administer fluids and begin antibiotics de watching weight. He do not touch child call ent urgently, and we have a whole Yeah. Okay. Almost unanimous. The world up guns. So should be hopefully fairly clear that this is, um, acute epical Titus. Um, well done, everybody. So I think the answer that they like to test this and, like, answer is always do not touch the child. Do not distress the child because I could just, like, close the air like that, calling anti urgently. You suspect it? Um, that's basically oh, and and I know usually they put Strider in the system, so I kind of threw it out, just so it wasn't as obvious as well, but actually stridors usually kind of a later side as well, So but the main things will be a low next insufficient like aged group, you know, restless, especially drooling. Um, and they'll usually lean forward because they find it a bit easier to like. It helps him breathe a bit more. So those things, right? So Okay, some quick questions now, um, rectified. Fresh number one. A 30 year old man presents with two day history of severe sore throat. He's not able to eat a drink. He rises. Any strange A list Drooling and Perenchio difficulty breathing in. His voice is hoarse. That's what it looks like. What? Your diagnosis. It's really quick. There's there's no poll, just type. Okay, So, um, we'll pause there and actually so well done. Those of you who thought supik a tightness, this answer is actually superb. Otitis, um, in terms of yet. So I'd say it's just in terms of the history on the fact that you're looking into the mouth on it Looks pretty normal. There's no other kind of history associative like in terms of like, uh, it would suggest other diagnoses. Um, but yeah, they probably test this. I thought I throw this in here for, like, fun. Okay. Censorship of the tightest next one. Um, I'm not very original. So here is another very similar statin. But that's the point. So here's a 27 year old now with the two day history. Severe sore throat not able to eat a drink. But now there's a large swelling on the right side of his Mac. He's drooling Parexel. He can't move his neck to the right. So what do you guys think? Yes. Okay, um, that is exactly what you should be thinking. So even though the examination of the throat is normal and the history is pretty telling that there's it sounds like some kind of deep next base infection. So just things that looking out for Okay, So spot diagnosis number one. What is this guy's Yes. Okay. I just looked at the 10, so I try to move a bit quicker. Correct. So that's a Chinese meet a tightest media with effusion, and I haven't gone over any of the ear stuff. Um, but I recommend just a quick look over a tightness, external media and kind of inner ear disorders, But, um, but it's good news. All this note the ear, but your bubbles Arab ogles. Um, And then one more spot. Diagnosis for a T, and we will move on. Yes. Call steatoma. This is called stable. I can't do anything except for it. Yes, that is just what it looks like. Have a few little images and a peak. Okay, so, watching on, I'm so sorry that took so long. Guys, let's move on to ophthalmology, and I'll try to be a bit quicker. So the first question is, um, 25 year old male presents to your clinic with new red left. All right. He describes This is a dull ache with some blurry vision, but there's no discharge. He said he had a similar episode in this right eye a few years ago. Um, his vitals and his examination is that let you read? And he has a past medical history of being diagnosed with Crone's 18. Nice. Yeah. Yeah. So the answer is anterior uveitis. I think it's in the question stem. And it just falls into the kind of cloud of, like, HLP 27 kind of associated diseases, which I'm sure you guys know all the rest of explained, like arthritis. All that sort. Very good. Uh, okay, that's answered yet. Okay. So, question to 35 year old Scottish woman presents to your GP practice with a four day history of vision loss. She gets pain when she looks around, she looks she feels otherwise fine. And she described a similar episode of a while ago, but it resolved average for three weeks. She didn't see a doctor that on examination, she has a relative afferent people defect and reduced visual acuity in the lab. Um, one investigation will help you with this diagnosis. I had a peak where I should be. Okay, right. So most people went for a week, and yes, that's correct. So, uh, Hopefully, what you picked up was that the underlying diagnosis is, um uh, ask. And with these medications, like with their kind of eye disease, other things you can have is like red desaturation. They might also percent with intraocular off the milk. Hoga optimal. Normal plegia. Um, and usually with these patients, where they look like is that the adoption in the affected I will be decreased on down in the side of the lesion and your other eye. Um, you'll have, um, exact site nystagmus at the same time that you're trying to. Yeah, I Yeah. So I'm a righty. And your answer. Moving on. Okay, so question three Elderly gentleman presents a me with one day history of painful red. I associate with blurriness of vision on examination. Has his right eye is red with people size that dilated mawr than his other eye thing. Intraocular pressure is 65 millimeters for mercury in his right eye. So what is your most appropriate response and right? Most people went with D, and that is correct. Correct? Correct. So hopefully most of you guys have picked up that this is acute angle closure, glaucoma. Um, and it is an ophthalmology emergency and bookmark this question because I'm pretty sure it's pretty consistently tested. I feel like on And, yeah, just know the management. I'd say, um, Breckel eyes is them and just know the management. Okay, so here's a slight on. Look, Coma won't me through it just now, but I'd say yes. Focus on, um, acute angle closure as a kind of type of question. They might ask about it, but it's worth knowing about of background about glaucoma, anyway. So here. Okay, so here's a slide. Just the basic features of knowing the difference between dry and wet macular degeneration. There Sometimes questions about these, but they're fairly straightforward in terms of, like, you know, trying to like the different candidate, the categories of different features of both of them. So that's I'm just finding out up to go over. Okay, but we're going to move on. Eso an 80 year old male presents with Saturn onset loss of vision in this right eye. On examination, he can only see, like in this right time, his six out of six acute in this left eye. No other symptoms or visual deformity arm is a past medical list of poorly controlled type two Delta diabetes in colleges. Um, his examination is bus. And what has caused the symptoms? Yeah, right. Nice. Nicely done. So most people went for B, and this is correct. Eso this is victories. Ham itch. That happened as a result off diabetic. Right now, they're just diabetic eye disease. Um, did do next eight next slide. So and there's a few things worth going over here on the slide about diabetic eye disease. Um, in that they like to ask stating staging questions. It's quite easy to ask a swell. So just know that you know, there's not a proliferative diabetic retinopathy know the different stages. I seem to remember Westchester where they were, like, you know, this person comes in, you have a look in there, I and they have, like, exudate and cotton wool spots. So, like, how do you stage this? So I would say definitely know that the differences between these three know but proliferative diabetic retinopathy don't jump the gun. They like to test nonproliferative like, as in this Lots of information here to test, um no, the management and kind of recognize What about our business means happy. Okay, You have anything else and see? Nope. Okay. So another really easy question for them to ask is the difference between, um central retinal vein occlusion and central retinal artery inclusion? I think this is like an easy point. So if you look to the left at baited collusion, they usually describe this as like stormy sky. Or like a margarita pizza. My favorite, which kind of makes sense like And then if you think of it as a baby and problem, there's issues with drainage trying. So it looks like a mass. It looks like pizza. Um, whereas with the artery occlusion, you're thinking it's a problem with supply. And that's why it's a lot cleaner, because there's not, you know, it's That's how I remember it anyway. But more distinctive is your like, cherry red spot, just like a pail appearance of the eye. That's how you tell them apart and usually the past medical history, like well, usually throw it like thing. But it is a vascular disease that they're gonna have vascular risk factors. So just remember bad we differentiate this from retinal detachment. Good question. Um, I'll be completely honest, art. I'm very sorry. I don't know if that's horrible. I'm getting this, like, I don't know. I did have a look into it yourself on, But maybe ask your friend. Um, I can ask them here for you guys if you like, but do not know. So I apologize that yes, they will see the funds. Do you guys go? Well, that everybody All right. So, um, I think it's really obvious what the issue is here, actually, eh? So this is just a ps a, um, To go over your eye policies. Um, I won't go over them just now because I think they're fairly straightforward, but just, um, be able to recognize them. Kind of know what they look like, just like in that up. Okay. And what infection are your risk of from wearing unclean contact lenses when you guys think All right, solid. So the answer here is actually Camp Amoeba. So that's kind of like the kind of classic one to think of when you hear about, like, unclean. Oh, my God. I thought I fixed that so unclean contact lenses. So that's gonna be kept me, But But I think the question stuff like carrot tightness is is very solid. Answer is well, actually, usually more to do with, I guess. Why does he have, like, trauma to the I was trying to remove the contact lens. I think I have. Yeah. So we're going along the same lines. Usually, if the question stem asks for, like, they show you this like they show that kind of, like, classic this, then. Okay, switch eyes, but basically Okay, uh, we'll skip this, basically is. If you see the kind of classic appearance of then dreaded ulcers under, like, pleuroscopy, it's going be herpes simplex Keratitis will be different questions, and then they can't amoeba. Hopefully crazy. So one last spot diagnosis from a mirror. I think this one's a bit mean, but all right, the other takers All right, Correct. So Well, they just kind of quickly. Okay, but, um yeah, so with the spot diagnosis, correct, it's retinitis pigmentosa. Um, and usually these patients that there might be a stem about them having, um, night vision loss first. And then because it kind of affects the peripheral retina results in peripheral vision loss as well. And then when you have a law, look, have a look at their eye. This is kind of the classic appearance on Do the words I'm not used to describe. This is kind of like black bone stick. You'll shipped pigmentation of the signs and kind of a modeling of the retinal pigment a thelium start to see that. Um, I don't know if you see that mouse. Um, yeah, so you can sort of see that at the edges. And there's nothing quite that looks quite like that. So because it congenital. Oh, I don't know. That's horrible. I don't know, actually. Sorry. I apologize again because it's not. My slides haven't gone over this in a while. Um, but yes, I think you know, good colleagues. You're good called still the answers here. Right? Sorry, guys, but anyway, and so that's it for ophthalmology and moving. All right, Now, to be the time it's moving along now to the last kind of 10. 15 minutes for dermatology, if that's all right. So right. I know I'm leaving a lot of questions on my desk, but hopefully just quick overview over things anyway. Okay, so hopefully you remember this man? Um so I'd say with dermatology, it's kind of similar with the rest of, like senses and that you want to go for, like, mostly pattern recognition. Um, and you know, you'd probably get away with quite like I don't want to say superficial, but kind of like superficial but wide, broad knowledge of dermatology and sometimes, like that might just be like if you have the knowledge base already that might just be sitting down with, um, going on a professor recent website and just to kind of go over everything, Um, and I just have a golden oak over everything, but just, like, have a scroll through a lot of images and just get acquainted with you probably see you like one of those images on your test, that kind of thing. Um, so they like to cover skin cancers. And as I'm sure you guys know, there's like, the Brady in between kind of last likely to metastasize to more like it metastasized. So I'd say, you know, this and obviously at one end of this is basal cell carcinoma. So, um, with your dermatology things, I'd say Just be able to recognize images and also buzz words. So four basic cell carcinoma, but they like to say it pearly. It's kind of like described as a crater, um, can be ulcerated, but they generally just less like gunky disgusting looking that SCC's, um as with a lot of skin conditions, usually in sun exposed areas, you might see a little weak blood vessels. Uh, so one step about this is actinic keratosis. So this is known as a precursor to develop into acid. See, So they look, they look more like this. They're bit cross tier. Um, yeah, so that might be a cheap angel. Quite saddle sometimes and just in areas of sun exposure. And then one step about this is bones disease. Um, so actually, the bones disease is a little more likely to turn into a see assess a CC than, um actinic keratosis. So it's just worth knowing this is well, and then you have full blown to play. That's cell carcinoma. So these guys do look a bit like master, so they usually ulcerated and it just looked a bit more raw. Um, can be nodulars well, areas of sun exposure. As we said on the key bit about this, as you guys know I'm sure, is that they're invasive in the capable of metastasizing, so you have to catch. These are have a low index of suspicion to catch thumb, and it's just you figure by excising them basically, um and then there's melanoma. So, um, I'm sure you guys know your a b c d e z about it in terms of like, the shape and stuff, um, shape regularity, you know, size of this. Been any change? So these do met the task sized early, so they're very important. That captures. Well, um, and what I'd say about these is that one don't be caught up. If it seems like they have a question somewhere. The the patient is quite young, like, say, of this 23 year old woman comes in, I'd say, Like I remember, that question is still the answer was still melanoma. So don't be thrown off by age. Um, and two, I think there may have been a question at some point about like like which of the following features suggests What did I say? Which of the following features means like it's more likely to like? It would suggest severity or suggests more likely to metastasize on. I think the answer was depth. So that's related to the Breslow thickness. So that's kind of going to give you your, um your kind of indicator about how far it's about. It's rather than size. Like how much has changed over the last six months? It's more about death. So Okay, so he's a side about some more skin conditions that, um just have a look over them. They're benign, Just like I've always no. Okay. And you guys, I'm sure have seen separate keratosis. So that those air like walks he stuck on appearance. Kind of lesions benign. Nothing to worry about. And Okay, so first question, um, spot diagnosis for this, honey, just think. Yes, definitely. Activate characterises. Uh, yeah. So this is actinic keratosis on the index finger. Just there with a bit of solar lentigo was blocked. Next. Um Okay, So what's this? Yeah, BCC correct. BCC. All right, So next question a Scottish six year old man presents with a suspicious lesion on this arm that has the following appearance. He's previously worked as a labor in Australia. What is this her that yet? That's melanoma. Trying to trick you guys here. Very good. Okay, Spot diagnosis again. What Is this okay? Yeah. Sounds like most of people have it. So yeah, that's acting. Dose is 9 lbs on. What's worth noting is that it's associated with BCG, diabetes and malignancy. So good. Okay, Another question. Eso a 27 year old female presents to the cheap. You with a six week history of diarrhea and abdominal game, mostly after meals on examination. She seems fine for abdomen is soft nontender. And she has this metrical itchy rash of capsules. Okay, guess we're ditching the poll. Uh, yeah. So, um correct. So this is celiac disease. So and, well, this is dermatitis herpetiformis rather, um, secondary to see the activity. So hopefully you picked up in the staff that she's had abdominal pain, especially after meals, and she's anemic. So I'd say focus on the question, stem that classic history. I know that she's anemic and she's got a rush, sir. Yeah, All right. So five year old girl has a long standing history of Exelon, and she presents a me with 48 hours of fever and itchy blisters over her eyes. Um, she's been to private school for the last week, and she's had contact with other Children. Um, her mother has been treating her recently with a cold sore. I looked last week. So what's your immediate management of this patient and otherwise is 12345 kind of kind of bothers me. All right, so I think that, you know, So that's right. So the the answer is, uh, you are five. Admit and give acyclovir in broad special bench bodies. And basically they need ophthalmology. Refuse. Wow, Eso This is dermatitis for pet com. They usually kind of present with, like, fever and clusters of itchy blisters are punched out. Erosions on the secondary. Bacterial skin infection is going to be treated with systemic and politics. It's okay to go bit overkill with this one on. Um, if his eyelid or I'm old and suspected, you have to consult ophthalmology like you don't fast around with this one, right? So almost a day and a promise. Okay, so I didn't know this one into a question, but basically, this is also another one to bookmarks. I think they like to test like there's a view of random things that number likes to test, and they love their matter of my Sinus Little stories are Okay, let's backtrack back. This one. Oh, um, what is the infection? And I'm not sure what exactly. Like, I think the point is not so much like what is, like the organism type of infection. So, um, it's kind of more than you know that there's going. There's some sort of like herpes infection going on. So it may be down to the question is it's herpes, basically, so that in the questions time, she's had a cold sore and she's been like playing with other kids. Um, so she has. Herpes is well, and there's also a skin infection. So you're treating with a cycle here, and it looks all right. Yeah. So I would bookmark this one because they like to ask if I think it's also a fairly easy question, Like an easy point the ground because, um, you don't need to know too much about it to a police. Understand that it's dramatic myositis you're gonna have, like, an autoimmune picture. You're going to have a rash, and you're gonna have muscle weakness like it's in the name. So it's not like it's not like, you know, two complex here, but, um, yeah, just know the other things as well. It can be associated with malignancy. Just know that's haven't idea of the investigations. You don't ask, but they're all kind of autoimmunity sounding like, for example, like Anti Joe one. You know, like anti me to like you know, all those things. But you can also ask for a skin about impossible to confirm the diagnosis. And here's some pictures off so that there could be some cuticle involvement they might tell you with a picture of their pulse. But ultimately, it's just having, like, look for, like, rash and muscle weakness and conjected in conjunction. Okay, fine. Okay, it's another one that they sometimes like to ask about is like in Planus. So just remember your six peas eso like black purple parietal political plane on popular And, um, it's been running, and I think, and they talk about what comes straight a which I don't think it's that kind of celebrity. It's more like sometimes it's describe is like Lexi a bit, too, but I'd say it's worth having a look at different pictures of it, because you can think you can try to remember. It is like a lifting but it actually shows up on different surfaces. So, um, just be familiar with how it can look at different parts of the body, and that's autoimmune. And you treat it with steroids cool. And another one that they sometimes ask is chorea. And, um, it's a pretty straight forward history kind of stomach. Well, on days just kind of asking about the different possible causes. Just know that that can be very rod and that you just typically treat these with anti histamines. You should avoid whatever calls them to kind of like pop. You try a short course of prednisone, give them an EpiPen if there's a really involvement, but fairly straight for it. Yeah, on. And, um, also that, uh, yeah, so here's some pictures of what it might look like. So just rash is caused by, like, some kind of precipitating factor. Okay, And here is just another slide to kind of go over like, you know, just another helpful side if you want to like screenshot everything. Um but as I say, like, properties has a fantastic collection online, and there's a lot of dermatology. There's a lot of topics. To be fair old. These three categories that are, like, hard to cover in more detail and just like an hour. But I'd say it's worth going over these, um, you know, first line here is like emergencies, Emergencies GP, you know, and as you guys, that with, like, acne, like, it's your standard. Like, if you if you give something, just know that kind of like a ladder of like, Oh, if they have this, have they tried an antibiotic cream and has it works? If you do give them, I scratch no one. Just know that they need to have, uh, two types of contraception if their child bearing age and your total scabies. I think over psoriasis. But that's where I'm having a P cat. Okay, so that's like the lecture. Basically wish you guys good luck. You'll be fine. Just take care of her size. Take care of yourselves until then. And I'd say, Even if it seems like even like I've lived through and or known people who have gone through their versions of worst case scenario, everything turns out fine. So you'll be three. It it. Thank you so much. Chelsea. Um, I just put a feedback form in the chancellor. If everyone could take a minute to go and fill that out, that would be great. Really helpful to Chelsea. Um, two years is, uh, PSA's. Well on, then I think Oscar is around. So, um well saw seven minutes fast. Thank you. Tell. See. Thanks, guys. Good luck. Just you. And I said cool guys and back, right? It's day to revision. Everyone's knock it, But stay with me. We're going to do one more thing and I'll let you all go and have lunch. Okay? She a shared my screen. Um, today we'll do hematology oncology, palliative care of breast cancer. Hopefully in lesson often hour. Because I'm really like when I did this, my brain was fried afterwards. So I'm not surprised that you're doing well if you're still here. I know friends who definitely bailed by this point. Right. So, hematology oncology, these are lower slides, unfortunately, lowers covering for someone later. She can't be here, so I've stepped up to the plate. Cool. So we'll start hematology. So these are the things that most usually you need to know about hematology, Um, was a from ah, tick disorder, See or venous thromboembolism. So these are the risk factors age, pregnancy gain, cancer. Having some drugs, you know, don't do drugs, kids. It's a bad idea, but sometimes we need the, uh, I'm being a mobile and having a family history or venous thromboembolism. I'm gonna try and not talk quickly, as they did a lot. Start the last talk on course. So some from optic disorders. This is the most common from a failure factor. Five. Leiden eyes inherited in an auto most dominant fashion. It presents young. They have recurrent from bumble is, um's. They often have a family history of that because it's also more dominant. You end up with all of the lovely embolize that you go to Lowe's of different places in the body. The basic treatment is you give them an anti coagulant. Be that warfarin or a dose pack on. You. Give prophylaxis only if they're pregnant or if they're gonna go on your surgery. Cool. This is another from body disorder, which can be acquired where antibodies target your own. Cell. Membranes often occurs with Lupus and some other conditions, but Lupus is the most common one. This's the one that is sort of obstetrics related where you get recurrent fetal loss is and you can't really explain why they have a thrombocytopenia and then you do a couple of investigations and find they have a prolonged a PTT and a prolonged clotting on a Lupus anticoagulant. Um, so if they have a being a strong believable is, um, your your anticoagulations. Other than that, you actually do give these guys prophylaxis with aspirin and smoking cessation advice. If it's super severe, which it can be when you get venous and out here or from versus all the time, then you would give them something, you know, suppression. This is a revision time table with a provision time. People are revision table off some of the other bleeding disorders that are inherited hemophilia and be do come up with the exams and they do have a bit of on willebrand on willebrand is autism one dominant inherited. It's partial or total loss off on willebrand factor on you end up with prolonged beating reduced factory on the treatments that they're on the right. Cool last that I will ask questions as we go get two questions. What you guys are answering. So if you have questions please save them to lend or later. These are some of the acquired being disorders. Feel free to screen shop the one that you need to know. That is the I see that can quite often happen, which is the disregulation between coagulation on federal assistance so yet costing and depletion of factors. And then you get bleeding as well. So it's a bit like you're bleeding and costing at the same time. Your body doesn't really know what to do, so it starts shutting down. You get a rash, and you need to treat the underlying cause. The underlying causes often like organ failure. War like platelet. Yes, you just organ support. Basically, this quite often happens in patients in I. T. U Phone, and then these are some Other required is always be realistic area. Extend your arm, and sure, you go to the way it happens in Children of it. It comes from E. Coli toxin mediated hemolysis and can cause renal failure. So you need to know about half arenas are That's quite fun. Question where they talk about hate for us with renal and hematology in your like what? That it's probably hate us. See need to look for somebody coli from somewhere. See, that's the treatment there and then hate. I hate 80 years old is a common one that does get talked about when you give people happen and you give them the number cytopenia cool. Eso anemias are a nice common one that like to be talked about. I'm not going to talk about Hemoglobinopathy because they literally just It'll be one question and there's no point me going into a really big detail on one thing. That could be a question. How is I ended? Deficiency anemia? That's a common question. It does come up so after you could just get blood loss be that menorrhagia rectal bleeding. Or so I'm making up a poor. You have a poor diet. You don't have enough iron in your diet. You were you present with fatigue, shortness of breath, palpitations. Um, they'll tell us. And then the two port ones are atrophic. Let's itis and angulus over like this. That's what you get with a deficiency. Anemia. Okay, So investigations that you were doing for on deficiency anemia is you check the ferritin and they're transferring. You don't really look a show. My and you can. It's all part of the partial when you do, The investigations on track with a ferritin they're transferring are the important stuff, so the protein is low. The transferring it's high because of transferring. Trying to scab insures iron and the ferritin's telling you that it's not been on line around. So what you do is then you could have an upper and lower jaw and just see if you're really worried or they're old. You do in urgent one looking for probably cancer. You can do a celiac screen as well. The management off Microcytic anemia is is you want to treat the bleeding, and then you give them some ferrous sulfate until the full don't count is normal, and then you get it three months off of that to replenish the stores off iron in the body. So the next one is a normocytic anemia. So normal city anemia is most commonly caused by chronic disease, so crowed cancer infections that last longer than 10 days. If you did an iron study on these people, the ferritin will be normal. That transfer it will be low, most importantly, so it's no iron deficiency. You're looking behind. A deficiency of transferring has to be high. You do a full blood count and infection screen. You consider looking at blood film to just see if there's anything on the blood from, but I probably won't be too much. Another cause of normocytic anemia is bone marrow failure, secondary to some of these things of malignancy drugs. So chemotherapy is quite a big one that causes bone marrow. Failure on did investigate, that is, you'd look at the blood and go What the pancytopenia wonder whether it's because the bone marrow is not working and not producing. So imagine for current diseases you just treat the course. So they've got cancer then, um, cancer. That thing that says Do not transport. That's not necessarily true because there are patients that still get transfusions. So there are patients with esophageal cancer who have chronic anemia normocytic anemia who do get transfused when they when that he would go to drop because they bleed from there cancer and they can't do it so they do get transfused. If they have a normocytic anemia, that's not necessarily right. Sorry on bone marrow failure. You transfused so macrocytic anemia. The commonest ones are B 12 and folate deficiencies. Which body needs to produce all of the blood cells ever be? 12 Deficiency is caused by some of these things. Gastrectomy is quite important. One and die. It's quite an important one. You and up with a bit of confusion gets sore tongue. Get a macro cytosis, and you can get some dorsal comments. Um, two columns. Symptoms. If it's really bad at a folate, deficiency can come from pregnancy and methotrexate him. The most common ones that we talk about. That's why women, when they're pregnant, get pro. Get fully on. Did you take methotrexate? You also get public so they end up with near psychiatric stuff. Confusion. Baby's getting your tube defects of months not caught enough fully floating around. You can have a pancytopenia, and then there's a macrocytosis investigations. Why is it like checking your B 12 and your folate and your PSA check and anti intrinsic factor antibodies? Because that's important for creating for scavenging B 12. The management is you replace the B 12 1st. That's important because it gives you a special something called Mobic. You combine degeneration off the court, I think so. You give the beetle first, and then you replace the phone. Lazy slug. Um, with, um, the B 12. And then they could have early after that. That's the one. Cool on then. More laxity. Anemia. Kamala cysts from those intrinsic and extrinsic causes the most common one for hemolysis in tract. Intrinsic wise is sickle cell anemia, and then the extrinsic wise is often infection or metallic heart valves. You get microcyte to strap macrocytic anemia. Yet regular reticulocytes doses split it regularly but jaundice. And you just check that for blood count bilirubin. The Coombs test is pretty important if you're looking for intrinsic stuff. Uh, I'm sure you bumped into a cruise test on pass Met. There's quite a nasty question about that. Somewhere in there. Yeah, You're just on this, like so management. You treat the cause you transfused as appropriate. And if it's auto immune that you give steroids a bit of a municipal essure or take away the spleen course and ask the summary, that's probably quite a key table that you probably should know. You have some arrows off what the MTV is doing. So the anti view would be low here. MCV will be normal here and then see be would be high here so that that's common ones. So if someone asks you what the cause is of a macrocytic anemia, you just go beetle folic deficiency and demolish this bank. Next schools is some scenarios which I don't know the answers to because Lord made the slicer, so legacy in here it ology. You get your leukemia's Angelin Foma is get acute leukemia and chronic leukemia. I always get really confused in my head, so it's best to just, like think about the lymph acidic ones car in the young child. They have a lymph lymphocytes. Have a seat in them. They're four Children, so therefore, it's a childhood religious. See, it's a really stupid idea of remembering it. But then my acute myeloid are middle aged people, so it has an eminent so happens in middle aged elderly people. And that's the acute stuff past my simple way of remembering. And then the key thing when they look at the blood film for acute myeloid is that you'll see our roads. I will say that the question If you see that you go right, that's a chemo I love to keep your next question off I go, Uh, sort of a day of these people often get chemo cysts on their off them. Very sick. It's quite sad. The usual ones that go to that's your zoster or two that I've seen the same department in tickets this place a chronic leukemias to lymphocytic once happening, elderly people. You don't really do about anything about it. You sort of just said they just have high white cell counts when you call, Uh, and then the Coreg myeloid is You treat that That's the one when you get in matinee, um, in matinee, which is your treatment because you've got that Philadelphia chromosome. That's the fancy one. That's that's This is the one that you need to remember. Um, because that is that this is the treatment you need to know. Okay, is your lymphoma's the Hodgkin's and no, no trans project has happened. The Hodgkin's stem for the question is, will be a young man with painless, asymmetrical lymphadenopathy, which is usually in his neck. Somewhere on, then the non Hodgkin's is on older, middle middle age patient with going aggressive, course burning, indolent course of symptoms. Uh, and these guys can get beef set the symptoms as well. She's like sweating and flushing. Investigations would be, as per that. You do some imaging to see how far the lymphoma has gotten you to aspirate their bone marrow and take a traffic biopsy from the hip. The best way to diagnose a lymphoma because it comes from a lymph node is that you would do a whole lot of biopsies. So biloba is another common hematologic away. Malignancy of the features of myeloma is you get crabs, low hypocalcemia, real failure, anemia and some bony pain. The questions, then, will often be an elderly man. He's tired, He's got bone pain, He's peeing a lot, and he's confused, so you'd be like, Well, interesting. So you do some blood to be taking calcium. Most importantly, and these are the ones that you need to know you for. Myeloma, this one in particular, is the one for my lung. Is that Bench Jones protein that you check in the year and you do a skeletal survey and a CT MRI to see how far it spreads as you would with most cancers, and you would get a bone marrow aspirate in a traffic biopsy to get a minute of diagnosis. Management of myeloma is often a strong it. They go into remission but do well for a bit, and then they relapse. Eyes, unfortunately, incurable, but treatment is often given. You can get allergenic stem cell transplant. You get some chemo and you get some other systemic therapies. And she goes, I need to know about so transfusion and actually needs a common these come up. I've seen a couple of these happen. This one's a common one. You just get a bit of a fever, so the question will be like this person's had a fever. What do you do? You stop the transfusion. You give a personal, but they'll be well, they'll be obsolete, stable. Everything else will be fine. So you just stop the transfusion. You give him some paracetamol and you restart the transfusion. If they remain stable, if they start destabilizing, actually abandon the transfusion and stop resuscitating. Yeah, mild allergic reaction, which you do exactly the same thing you stopped. Transfusion. Give that test means you're reassess, so it should be a transfusion reactions. These air bit rubbish. Um, with this one, the patient will be unwell, they'll be hemodynamically unstable. Uh, you need to stop the blood transfusion now and start resuscitated them straight away. Well, these guys will be sick. I've seen this happen once on that was like a big incident. It was crazy on our fax. This is your usual symptoms about actually what you guys will on that. And you got a trolley on your taco. Your trolleys, when you get dyspenea and hypoc CIA within six hours, it's not media. That's later on these arm or later on down the line. Yeah, if you have hypertension and you do a chest, actually that have bilateral pulmonary empathy infiltrates it stopped transfusion, recess them and give them some oxygen tacos when you given too much in too fast times a pony DMX all flooded to their lungs So they get your breath and that he is genotype and shooting. This is common in patients who like how heart failure have had a load of fluid already. So you would resuscitate diaries and give them some oxygen. That's even college E. In a nutshell, I would go online about blood products. That's important. So effort pee Cryoprecipitates. Yeah. And your red cells anticoagulants like it does. Actually, warfarin did not cover it Here. You guys can go and quickly learn about those on. I would know how to reverse warfarin is Well, a question that has come up every single year is the reversal of Wolfram. So you need to go look it up right in college, on policy. So we'll touch on emergencies only pretty much because the other cancers will be covered other way elsewhere in other talks because I don't want to bore you guys. So a 67 year old man has me experiencing some back pain for the last series. Losing control. Strength of his legs were going to lose control. His bladder got past millions. Your prostate cancer. What's going on here? Take it in the channel. Oh, yeah, He's got metastatic spinal cord compression, which is what, a tumor or a collapsed vertebra body from a tumor pressing on the spinal cord or the quarter a quarter? These patients turn up with pain weakness, bowel dysfunction have your e problems, and they often have the peri a paraneal loss of sensation. That's a key thing in the question, which I don't usually get. This is common stuff, and it's come from breast cancer, prostate cancer, myeloma, a lung cancer. I think you do for this is you need to do an MRI of the whole spine to see where it's impinges. You give them some dexamethasone on a PPI. Could you could give him the PPI because they're getting steroid. The definitive management is your phone, your surgery first and say, I've given some stories in the PPI what they do and they go, let's have a look at the MRI and they go, Oh, okay, Maybe we'll operate or they'll go know just getting ready. A therapy. Um, usually these candidate. By the time they've got on methotrexate Vinyl compression of the cord, they're creeps. Poor surgical candidates, so they'll get radiotherapy. Um, expression is a 45 year old lady. Three days of feeling of it, rubbishing feverish. She's got breast cancer, and she's had chemo nine days ago. What's happening here? Yes, correct. This is a neutropenic sepsis. So neutropenic sepsis is an infection, which you get more than a week after systemic chemotherapy. They turn up with parks. Yeah, they've got symptoms of a source, so be a cough. Urinary. Want something else, I'll feel naked. And if you do, their blood's than you. Their neutrophils will be another will be nowhere used cars and lymphomas and leukemias quite often, but can occur with any cancer. So investigations wise, you do a thorough examination, looking for the source. You take their blood's your culture. You do some viral throat swabs he did for the year, like you would with any septic patient of these guys. Just they do well if you get to the fast, so you have to manage them appropriate with use. And the way you do that is you give them a time to politics as you would with any accepted patient. But you guys need to know that it's tires 4.5 lbs. You have to know that you have to look at the question. Go. It's tired. It's neutral except this, and you move on. Thanks. Question certified old lady. In any short breath, she's got low saturations. She's swollen and the veins on her chest to distended. Yeah, this is Emily has said superior vena, a cave obstruction, which is when your cancer six on your superior really care about seeing this twice because that face is huge and you just get obstructed basically from the top half up. It's like you get a whole load of swelling in your legs, but it's all in your face and your neck. Get distended veins because they're trying to compensate for the fact that the big vein is not working properly. So the rest of your veins try and engorged to get more blood help. Uh, they get headaches, I get congested, it bleed on. They feel a bit rubbish. It's typically comes from bronchial cancers. Lymphoma? Uh, she's me. I have a cold. Uh, these people, any blood stream needed chest X ray on. They will often just get a CT chest in the first incident because you can see the specialist. See, Then you asked for a venous face basically on the radio for will look for the SBC. I have to sit them out. Right. Give us, um, Dex to try and shrink the cancer down. Shrink away the edema. And these people potentially need a stent to be managed. And they might need chemotherapy too strict. The cancer. Often if they get to this stage, they're not doing very well. And these patients don't last very long. I certainly on your mind brought in confused, agitated pains in his arms and his legs more breathless recently. More everything's going on here. Yeah, yeah, care. I really go high. Calcium got a ligament. High calcium, which is common very common with lots of cancers, which often comes for bony mets or the cancer, produces a P T H like peptide, which actually the same way as parathyroid hormone and causes the calcium levels in the body to rise. So they get a T high BP. They feel rubbish. They get a dull pain and constipation. They pee a lot. They're thirsty and they get fractures. So the important thing first is the bones. Last owns their groans on their psychiatric morning. That's the important stuff to remember the bone in the pain, bone pain they get. Kidney stones feel crap because they're growing a lot when they get psychiatric moods that confused. This comes from lung breast in multiple myeloma, too. So these are the ones that you do. You can do an E. C G, and if they're asymptomatic on their calcium is less than three. You give them some or fluid on you can also give IV fluid as well on you. So if they're asymptomatic less than three millimeters per liter, you just you can re hydrate with all fluids. You don't have to get my beautiful. It's if they're gone and a or all wrote, use it getting to drink just Martin. If they're symptomatic aware, constant greater than 33 liters via an IV line off sodium chloride and then you give them an IV is alendronate, and then you have to recheck that casting levels to see that what you're doing works So these other topics are not covered in this oncology talk. Lung cancer has hopefully being covered in your respiratory breast cancer. I'm going to talk about in a second job tract cancer. I'll briefly color because I didn't talk about it when I did your eye Common medications or you're like trust. Who's Mab? Your goes really in your like dexamethasone. You have a list on your knowledge, outcomes of what drugs you need to know about what we do and any side effects that you general Isis syndrome. I would go away quickly, read about. We got a question in that in our exam last year. Course is, that is that. And then if we need that done, do you that right? Within a blast through breast stuff, color of the stuff on a palliative care stuff? It's 1 30. I'm eating into your lungs. I'm sorry. As you know, we have Magical Pole, which appears in a second for you. To answer this question, a 76 year old lady has come to the breast clinic for the results of her breast biopsy, which is, er, positive PR for a PR negative and her two negative. She's gonna have surgery and radiotherapy. What is that Appropriate management after that? To get addicted, right? That's enough Answer in time to the correct answer here is they're going to get letrozole. So this they're 76 then, most likely postmenopausal Letrozole is a treatment for all postmenopausal ladies with breast cancer or who have a eastern receptor positive cancer. If they are premenopausal, they get tamoxifen upper until they come menopausal, and then you move them to Electra's all on. If her two positive, they get trusted him out. Did it do so? Breast cancer is divided into a little doctor and lobular breast cancer. You guys need to be able to perform a breast exam for your skis. It comes up. So I would recommend that you go practice with your friends. Which which Which bit did you want me to repeat again? Her two positive. You get them trust to use them up. If they're eastern receptor positive and they are postmenopausal, we give them electrical given tamoxifen in that pre menopausal. Okay, cool. So I guys, you need to do be able to do a breast exam on that. Involves asking a friend. Can I please examine your area? Exactly. Okay. So you'll get risk factors for breast cancer are being older, being female, being white, having a positive family history. You get eastern exposure either that from indulge in it or exogenous source being radiated for, like a cancer or something. Or you could have brought some benign breast disease. Breast disease is our cities with breast cancer. First cancers are founded by screening, but some coming after feeling in the normal mass again see the GP symptoms they could possibly have is a breast mass, Some discharge, some lymphadenopathy in their exhilarate, not some skin changes. Screening is offered to 47 to 73 year olds every three years. The mammogram. Your examination. You do breast. If you're younger, you get a breast ultrasound due to your breast density. There's any concerns about on your mammogram or in the examination, your biopsy, the treatment. The sequence of treatment is surgery, then chemo, then radiotherapy, then endocrine therapy and the consultant breast surgeons will decide that what they want to do it. We'll do it if they're different things. Surgical options are as written. There you do a sentinel lymph node biopsy. See if there's no real involvement. This is what I was talking about in the question of second guy with the endocrine therapy. I'll give you a chance to screen shot at all. Right that down because this will come up. This comes up every year was always a question on. Everyone gets that next. In a twist about it, you just This is what you need to come with me. So they are, you know, number two. 40 year old women. Breast discharge left breast. Only blood stained feels a rhyme. No symptoms. Both breasts are normal is a fixed tender lumps beneath the left nipple exhilarating. The tails expense have no masses. What's going on here? I went over this question. I did this with Boston. It, But I forgot on the home stretch. I think I got it wrong when I did this. Impossible. So the answer is memory duct. Act easier. You got it all written. See, don't go me on this because there is a cost. This is an exact question on password, which you guys can go and do. So it might be in my actually me. See, I wrote this question, then went away from a question possible and then didn't go back to so please a great woman. Thanks. So these are common causes of nickel discharge. Physiological curse doing breastfeeding. It can get blacked area, which is secondary to the drugs. Awesome emotional events. You get hyper prolactinemia, which often comes in the pituitary treatment in the question that probably have a bitemporal hemianopia as well. Um, memory doctor. Tuesday you get dilatation of the breast ducks, which is common in men approval pausal women and smoking. And they have a thick green discharge. Yes. Sorry. It is a drug for papilloma to the last question. I had a stroke when I was writing this cat so you could get blood stained nipple discharge. And they often have another lying mass for associated lymphadenopathy that goes on with it. Next question. 29 lump in the left breast concerned located below the nipple. Mild tenderness. No triggers or trauma. Well defined. Two centimeter Have mobile mask on the left breast. No skin discoloration of distress for the answer. For this is a fibro adenoma because it's well defined. It's mobile. You can have give a bit of a field, but they're well, there's no skin discoloration with discharge. This is a little bit of benign breast lesion. Geez, no, But if you want a really good resource for for benign breast lesions, we haven't really because when I got to this tutorial and heard of anything about it, unless lesions when I did this in here five or whenever it was. And I was like, What is a benign breast lesion? Because I just learned about breast cancer and didn't realize we had to learn about this is what so teach me surgery Teach me. Surgery is great for benign breast Asians. These are some of the common Bryant. Benign breast lesions advise that you take a picture off this table and go away on done it all bring it to the day. Yeah. What's the matter? Okay, well, who's doing Carol? So, a 75 year old man. Any generally unwell? Treated with chemo? Eight days ago. High temperature. Oh, is okay. Bloods taken. Results not available. What do you today? All right. And drink some water. It's a long day. Okay, So the answer to this is because we've done some teaching already on this, uh is it gives him some antibiotics and give the antibiotics before they get to the medical student. You know, sneakily tricked you. If you said medical receiving unit, you need to treat the patient before they move on. You can't just move the ball and let some other doctor there you are treating patient first, so we'll talk about colorectal cancer very quickly, So I didn't talk about that last time. The majority of our new cast know. Does that come from f yourselves and mostly come in the colon but can come in the rectum as well? These do the risk factors for them. You have to be you could be old. Often colitis is quite common on. And then there's two like familial syndromes. Slash mutations that comment The classifications for is dude fascination, which is divided into a B and C diagnosis they get. They have for the how often will have rectal bleeding, a change in their bowel habit that I have a mass somewhere or don't have anemia. Anyone elderly with an anemia that is enormous. It IC and normal chromic should be suspected of having colorectal cancer until proven otherwise. So these people needed surgeon within two weeks. Colonoscopy, if they haven't eaten back as a screening, is for 52 74 year olds every two years in Scotland, and it's done with the fecal occult blood test. If that's positive, then you get a colonoscopy. The colonoscopy needs some bowel preparations with elasticized because you can't say anything because the whole letter shit in your way. If they have confirmed colorectal cancer than asked, well, your usual malignancy stuff. I need a CT to look for nodal and metastatic disease. Cool saw I would. George earlier talked about surgery on, he said, Well, we probably don't need to know about surgery but actually they have bean questions in the past. Do you need to know whether surgery is? And I'll show you a nice little picture you can screen shot in a second? So this is cancer. It's stage 13 centimeters on the annual. But but in balls anal sphincter. Well, I make a stage 21. I really question myself. So So what? Surgery really benefit from ABC people? Yeah, the correct answer put this is okay, says. And because it involves a nose finger is an abdominal paraneal resection real. And the reason for that is you take the skin and the sphincter away in the Donald parent or a section It would be a lower anterior. Sexually, it didn't involve anal sphincter, but because it involves a know stricter is an abdominoperineal resection. These other sections, this is what they look like. They might tell you that the the masses in the ascending colon, and then you have to go right, which will collecting you're looking at or the descending colon. It probably won't ask you to take out the whole colon, but they will expect you to know like diverticuli. They expect you to know if it's in the sigmoid. If it's in the rectum, it doesn't involve the sphincter. You're probably looking at a lower anterior resection. It doesn't it is no in the rectum, and it's mainly the signaling that it's probably a higher interior section. There is a good question. There's a good couple of questions on this on past made that I would suggest you kind of look at so you would receptor most of the patients. If they're a bit bit past up the stage, they might need chemo radiotherapy. If it passed this age of that on surgery, then they might need stenting to alleviate the symptoms board. Need a diverting colostomy. So a man is here from his nursing home with unrelievedly pain in his body. Basically back out of COPD. So non small cell lung cancer with known bony mets attention, heart failure. You got bit tacky. He's otherwise well, some reason the necessary Give him his liquid pain. That's that's a bit from his community card, actually can see that he takes 30 mg arm off associative breakthrough pain. What should his breakthrough dose be? Yeah, Do you, uh, a low of the monkey? So the answer to this, I'll answer that low high anterior resection at the end of this, um, this talk because people want to go on a sleep s So that's this is, as most of your points out is, 10 mg. The way you work out for morphine dose is what you do is you calculate the full dose so B d is twice a day. They have 60 mg in 24 hours on the breakthrough. Pain is a six off that so that the dose for that is 10 mg the 60 divided by six. He's the one that does this screen shot this table learned this table. You will get asked about it in the exam. Uh, I would learn the conversions. Yes. Are, uh, so there will be questions about analgesia. Breakthrough pain is 16. It's not 1 10 ft 16 on the numbers will usually add up to see increments 2.5 to 5 mg because that's the dose that it comes in in the bottle. It won't be something wild, like 7 mg or 3 mg people. Be that you need to know the equivalent of a morphine. That's the concern version table there, they will ask you about that exam. I got a question about my my exam. It's not about pain. Lower ladder. Don't just go charging in with the morphine straight away. They might benefit from some paracetamol. Some height dihydrocodein first of all, side effects of morphine include constipation. It's very common these patients often get constipated, so you might need a laxative in order to get fill it sick, and they can become sedated on require reversal with the lock. The way to work out the dose of some cut morphine is tow half the dose off the 24 hours of morphine. So this person, if they were looking for some morphine in the last 24 hours you have 30 mg, not 60 patients with kidney failure get alfentanil reprint often, often to know. So see what if they got CKD in the question and you see what what analgesia they're looking for is probably one of those things. But also I guess eso a 90 year old female is admitted for the management of terminal in stage Mets Metastatic bile cancer. There were issues with constipation, nausea, vomiting, requiring a high amounts of opiates and just increase the dose. Nausea. Vomiting is now worse. What antibiotic would benefit this person? This is a basket of questions, which I suspect that some of you might know. Grab. I'll explain it. Go right. Here's the answer. Correct answer is don't paradigm. So in this question missed out metoclopramide on purpose. It metoclopramide was there. That's the answer. The reason West on paradigm is because it's an anti emetic effect. A prokinetic affect your looking to move the bowel. This person has gastric creases because it got nausea, vomiting, which is stopping the gastric going, going, going back the other way. You got constipation. That bowel is also got process. So if you have medical Promide as the answer, that would be the answer here. But because I missed our yeah, the answer is, don't paranoid, but don't don't particular dose. Give Cyclizine where there's a mechanical bile obstruction. Have not mentioned anything in this question about a mechanical bowel obstruction or raised intracranial pressure. She wouldn't give that here. Dexamethasone is using lots of different places, but the reason we had Dex in nausea is when they have raised intracranial pressure. It's not enough for it's not a first line for this case. Lever. Rumor. Promazine is a very common anti emetic that we use, and it's using palliative care. Bye. Subcontinent Action or infusion? This person still has an all right. It's not usually last used first line because it's quite sedative. Ondansetron can be used for nausea. Second treated chemotherapy is not good for this place on off on off. And with the Lancet on you get constipation and it would make the situation worse. So you don't want to give this person on down strong because they'll get more constipated. So the answer is down. Paradigm or metoclopramide is there, Right? Okay, I'll call you all that on purpose for fun because I'm a busted. Would she be on Libya lever anyway, as in the test battery Mets? Yes, potentially. But you probably wouldn't be able to would probably add that in later, given that she's still well, I haven't said that what he's got. Stage metastatic disease. But lots of people have in stage metastatic disease. No. Well, so you don't need to get charged him to leave by straight away. So you guys should know These are the antihistamine case. We care most you should know them and what they do, what they're used for. They often go in surgery. It was in the last days of life. You haven't seen one of these forms yet. Well, go and have a look at so in someone's who's palliative. You need to know what this for this thing, this form follows everyone around. This form form follows patients around wherever they can't be signed by a fire ones and must be can't signed by the supervising physician a consultant for it to be a legal document. Really? As the end of my talk, these are my top tips for passing exams. Don't panic when you're doing the exam. Know open question that shit. I don't know the answer. Take a breath and breathe and you got less. Move on is all our mentality exams. It's a dark place working in a room when it's slightly something outside or even like enjoyable. You have got this. You've been at it for six years. It's all about it's all the mental process revising prisms. I have to revise your gonna look down there with a bastard. I hated it. Past medicine is your friends Go and revise it. You should do a lot of questions. At least five, if you can. I advise that you want to get outside, have some fresh air I spend a lot of time inside. When I had a rising and it's stuck on the lower ground for the library was my place of worked, and I never want to ever go back there again. You spend time with your friends going to have a beer, have some dinner chat about shit, chat about life, child out who's doing what with who on used to, like, let go a little bit. Also, you can revise your friends as well, if you really want to listen to the big boss. Man blew the he knows everything. He will give you a revision sessions and just please go to his revision sessions. I went is revision sessions, and they will wicked. They were so good. He's brilliant or what? He does a bubble. You chose to do medicine. Enjoy it. It's great. We need to have a great degree, and once you get out a promise you it's no, it's actually pretty good about the other side, and you will get out of it. You'll be fine. Um, please. Feeling my feet back. I need it for my portfolio. Thank you for listening to my talk. If you have any questions, I'll poke around for that afterwards. Um, other than that. Thanks, guys. I hope you'll pass your exams. You're gonna be great. Thank you so much. That was, um, quite impressive. Especially considering you have 24. I was noticing that you've been thanks to, uh um Well, we have a bit of time left just at the end of the day now for a quick you and a So I think that Chelsey still online on board George should be coming back as well. Um, so, you know, through the mic and through the chart box. So if people want to stick around for that, I saw him was having lunch. But time now, this one was Just ask me about Hodgkin's slides, old circulate them. So growth. Um, from all that. She doesn't say you like them to be 10 days, Corinne. It's just a question on hypothetically, what could come up in Edinburgh skis